Buy BOOKS at Discounted Price

Algebra Of Vectors

Class 12th Mathematics RD Sharma Volume 2 Solution
Exercise 23.1
  1. Represent the following graphically: i. a displacement of 40 km, 30° east of…
  2. Classify the following measures as scalars and vectors : i. 15 kg ii. 20 kg…
  3. Classify the following as scalars and vector quantities: i. Time period ii.…
  4. In fig 23.5 ABCD is a regular hexagon, which vectors are: i. Collinear ii.…
  5. Answer the following as true or false: i. vector a and vector a are collinear.…
Very Short Answer
  1. Define “zero vector”.
  2. Define unit vector.
  3. Define position vector of a point.
  4. Write vector {pq} + vec{rp} + vec{qr} in the simplified form.
  5. If vector {a} vec{b} are two non-collinear vectors such that x vector {a}+y vec{b} =…
  6. If vector {a} vec{b} represent two adjacent sides of a parallelogram, then write…
  7. If vector {a} , vec{b} , vec{c} represent the sides of a triangle taken in order,…
  8. If vector {a} , vec{b} , vec{c} are position vectors of the vertices A, B and C…
  9. If vector {a} , vec{b} , vec{c} are the position vectors of the vertices of a…
  10. If G denotes the centroid of Δ ABC, then write the value of vector {ga} + vec{gb} +…
  11. If vector {a} vec{b} denote the position vectors of points A and B respectively and…
  12. If D is the mid-point of side BC of a triangle ABC such that vector {ab} + vec{ac} =…
  13. If D, E, F are the mid-points of the sides BC, CA and AB respectively of a triangle…
  14. If vector {a} is non-zero vector of modulus a and m is a non-zero scalar such that m…
  15. If vector {a} , vec{b} , vec{c} are the position vectors of the vertices of an…
  16. Write a unit vector making equal acute angles with a coordinates axes.…
  17. If a vector makes angles α, β, γ with OX, OY and OZ respectively, then write the value…
  18. If vector {a} , vec{b} , vec{c} are position vectors of the points A, B and C…
  19. Write a vector of magnitude 12 units which makes 45° angle with X-axis, 60° angle with…
  20. Write the length (magnitude) of a vector whose projections on the coordinate axes are…
  21. Write the position vector of a point dividing the line segment joining points A and B…
  22. Write the direction cosines of the vector vector {r} = 6 {i}-2 hat{j}+3 hat{k}…
  23. If vector {a} = {i} + hat{j} , vec{b} = hat{j} + hat{k} vec{c} = hat{k} + hat{i}…
  24. If vector {a} = {i}+2 hat{j} , vec{b} = hat{j}+2 hat{k} write a unit vector along…
  25. Write the position vector of a point dividing the line segment joining points having…
  26. If vector {a} = {i} + hat{j} , vec{b} = hat{j} + hat{k} , vec{c} = hat{k} + hat{i}…
  27. If vector {a} = 3 {i} - hat{j}-4 hat{k} , vec{b} = 2 hat{i}+4 hat{j}-3 hat{k} and…
  28. A unit vector vector {1} makes angles { pi }/{3} frac { pi }/{2} {j} hat{k}…
  29. Write a unit vector in the direction of vector {a} = 3 {i}-2 hat{j}+6 hat{k}…
  30. If vector {a} = {i}+2 hat{j}-3 hat{k} and vector {b} = 2 {i}+4 hat{j}+9 hat{k}…
  31. Write a unit vector in the direction of vector {b} = 2 {i} + hat{j}+2 hat{k}…
  32. Find the position vector of the mid-point of the line segment AB, where A is the point…
  33. Find a vector in the direction of vector {a} = 2 {i} - hat{j}+2 hat{k} which has…
  34. What is the cosine of the angle which the vector root {2} {i} + hat{j} + hat{k}…
  35. Write two different vectors having same magnitude.
  36. Write two different vectors having same direction.
  37. Write a vector in the direction of vector 5 {i} - hat{j}+2 hat{k} which has…
  38. Write the direction cosines of the vector {i}+2 hat{j}+3 hat{k}…
  39. Find a unit vector in the direction of vector {a} = 2 {i}-3 hat{j}+6 hat{k}…
  40. For what value of ‘a’ the vectors 2 {i}-3 hat{j}+4 hat{k} and a {i}+6 hat{j}-8…
  41. Write the direction cosines of the vectors -2 {i} + hat{j}-5 hat{k}…
  42. Find the sum of the following vectors: vector {a} = {i}-2 hat{j} , vec{b} = 2…
  43. Find a unit vector in the direction of the vector vector {a} = 3 {i}-2 hat{j}+6…
  44. If vector {a} = x {i}+2 hat{j}-z hat{k} and vector {b} = 3 {i}-y hat{j} + hat{k}…
  45. Write a unit vector in the direction of the sum of the vectors vector {a} = 2 {i}+2…
  46. Find the value of ‘p’ for which the vectors 3 {i}+2 hat{j}+9 hat{k} and {i}-2p…
  47. Find a vector vector { mathfrak{q} } of magnitude 5 root {2} making an angle of…
  48. Write a unit vector in the direction of vector {pq} , where P and Q are the points…
  49. Find a vector in the direction of vector 2 {i}-3 hat{j}+6 hat{k} which has…
  50. If | vector {a}| = 4 and –3 ≤ λ ≤ 2, then write the range of | lambda vector {a}|…
  51. In a triangle OAC, if B is the mid-point of side AC and vector {oa} = vec{a} ,…
  52. Write the position vector of the point which divides the join of points with position…
Mcq
  1. If in a Δ ABC, A ≡ (0, 0), B ≡ (3, 3, √3), C ≡ (–3, √3, 3), then the vector of…
  2. If bar {a} , overline{b} are the vectors forming consecutive sides of a regular…
  3. Forces 30 bar {a} , 50 vector {b} act along OA and OB. If their resultant passes…
  4. If bar {a} , vector {b} , overline{c} are three non-zero vectors, no two of which are…
  5. If a ( 60 {i}+3 hat{j} ) b ( 40 {i}-8 hat{j} ) and c ( a {i}-52 hat{j} )…
  6. If G is the intersection of diagonals of a parallelogram ABCD and O is any point, then…
  7. The vector is a Mark the correct alternative in each of the following:…
  8. In a regular hexagon ABCDEF, Then, vector {a}e = Mark the correct alternative in each…
  9. The vector equation of the plane passing through bar {a} , vector {b} , overline{c} is…
  10. If O and O’ are circumcentre and orthocentre of Δ ABC, then o vector {a}+o vec{b}+o…
  11. If bar {a} , vector {b} , overline{c} overline{d} are the position vectors of points…
  12. Let G be the centroid of Δ ABC. If then the bisector vector {ag}_{r} in terms of bar…
  13. If ABCDEF is a regular hexagon, then vector {ad} + vec{eb} + bar {fc} equals. Mark…
  14. The position vectors of the points A, B, C are 2 {i} + hat{j} - hat{k} , 3 hat{i}-2…
  15. If three points A, B and C have position vectors and y {i}-2j-5 hat{k} respectively…
  16. ABCD is a parallelogram with AC and BD as diagonals. Then, a bar {c}-b vector {d} =…
  17. If OACB is a parallelogram with vector {oc} = vec{a} and vector {ab} = vec{b} , then…
  18. If bar {a} overline{b} are two collinear vectors, then which of the following are…
  19. If figure which of the following is not true? a Mark the correct alternative in each…
Exercise 23.2
  1. If P, Q and R are three collinear points such that vector pq = vector a and…
  2. Give a condition that three vectors vector a , vector b and vector c form the…
  3. If vector a and vector b are two non-collinear vectors having the same initial…
  4. If is a vector and m is a scalar such that m vector a = vector 0 then what are…
  5. If vector a , vector b are two vectors, then write the truth value of the…
  6. If vector a , vector b are two vectors, then write the truth value of the…
  7. If vector a , vector b are two vectors, then write the truth value of the…
  8. ABCD is a quadrilateral. Find the sum of the vectors vector ba , vector bc ,…
  9. vector ab + vector bc + vector cd + vector de + vector ea = vector 0 ABCDE is…
  10. vector ab + vector ae + vector bc + vector dc + vector ed + vector ac = 3…
  11. Prove that the sum of all vectors drawn from the centre of a regular octagon to…
  12. If P is a point and ABCD is a quadrilateral and vector ap + vector pb + vector…
  13. Five forces and bar ab , bar ac , bar ad , vector ae and vector af act at the…
Exercise 23.3
  1. Find the position vector of a point R which divides the line joining the two…
  2. If vector a , vector b are the position vectors of A, B respectively, find the…
  3. Show that the four points A, B, C, D with position vectors vector a , vector b…
  4. Show that the four points P, Q, R, S with position vectors vector p , vector q…
  5. The vertices A, B, C of triangle ABC have respectively position vectors vector…
  6. Let vector a , vector b , vector c , vector d be the position vectors of the…
Exercise 23.4
  1. If O is a point in space, ABC is a triangle and D, E, F are the mid-points of…
  2. Show that the sum of the three vectors determined by the medians of a triangle…
  3. ABCD is a parallelogram and P is the point of intersection of its diagonals. If…
  4. Show that the line segments joining the midpoints of opposite sides of a…
  5. ABCD are four points in a plane and Q is the point of intersection of the lines…
  6. Prove by vector method that the internal bisectors of the angles of a triangle…
Exercise 23.5
  1. If the position vector of a point (-4, -3) be vector a find | vector a|…
  2. If the position vector vector a of a point (12, n) is such that | vector a| =…
  3. Find a vector of magnitude 4 units which is parallel to the vector root 3 i + j…
  4. Express vector ab in terms of unit vectors 1 and j when the points are : (i) A…
  5. Find the coordinates of the tip of the position vector which is equivalent to…
  6. ABCD is a parallelogram. If the coordinates of A, B and C are (-2, 1), (3, 0)…
  7. If the position vectors of the points A (3, 4), B (5, -6) and C (4, -1) are…
  8. If be the position vector whose tip is (5, -3), find the coordinates of a point…
  9. Show that the points 2 i , - i-4 j and 1+4 j form an isosceles triangle.…
  10. Find a unit vector parallel to the vector i + root 3 j
  11. The position vectors of points A, B and C are lambda i+3 j , 12 i + μ j and…
  12. Find the components along the coordinate axes of the position vector of each…
Exercise 23.6
  1. Find the magnitude of the vector vector a = 2 i+3 j-6 k
  2. Find the unit vector in the direction of 3 i+4 j-12 k
  3. Find a unit vector in the direction of the resultant of the vectors i -…
  4. The adjacent sides of a parallelogram are represented by the vectors vector a =…
  5. If vector a = 3 i - j-4 k , vector b = - 2 i+4 j-3 k and vector c = i+2 j - k…
  6. If vector pq = 3 i+2 j - k and the coordinates of P are (1, -1, 2), find the…
  7. Prove that the points i - j , 4 j-3 j + k and 2 i-4 j+5 k are the vertices of a…
  8. If the vertices A, B, C of a triangle ABC are the points with position vectors…
  9. Find the vector from the origin O to the centroid of the triangle whose…
  10. Find the position vector of a point R which divides the line segment joining…
  11. Find the position vector of the mid-point of the vector joining the points p…
  12. Find the unit vector in the direction of vector vector pq where P and Q are…
  13. Show that the points a (2 i - j + k) b (i-3 j-5 k) , c (3 i-4 j-4 k) are the…
  14. Find the position vector of the mid-point of the vector joining the points P…
  15. Find the value of x for which x (i + j + k) is a unit vector.
  16. If vector a = i+j+k , vector b = 2 i - j+3 k and vector c = i-2 j + k find a…
  17. If vector a = i + j + k , vector b = 4 i-2 j+3 k and vector c = i-2 j + k find…
  18. Find a vector of magnitude of 5 units parallel to the resultant of the vector…
  19. The two vectors j + i and 3 i + j+4 k represent the sides vector ab and vector…
Exercise 23.7
  1. Show that the points A, B, C with position vectors vector a-2 vector b+3 vector…
  2. If vector a , vector b , vector c are non-coplanar vectors, prove that the…
  3. If vector a , vector b , vector c are non-coplanar vectors, prove that the…
  4. Prove that the points having position vectors i+2 j+3 k , 3 i+4 j+7 k -3 i-2…
  5. If the points with position vectors 10 i+3 j , 12 i-5 j and a i+11 j are…
  6. If vector a , vector b are two non-collinear vectors, prove that the points…
  7. If vector ao + vector ob = vector bo + vector oc prove that A, B, C are…
  8. Show that the vectors 2 i-3 j+4 k and -4 i+6 j-8 k are collinear.…
  9. If the points A (m, -1), B (2, 1) and C(4, 5) are collinear, find the value of…
  10. Show that the points (3, 4), (-5, 16), (5, 1) are collinear.
  11. If the vectors vector a = 2 i-3 j and vector b = - 6 i+m j are collinear, find…
  12. Show that the points A (1, -2, -8), B (5, 0, -2) and C(11, 3, 7) are…
  13. Using vectors show that the points A(-2, 3, 5), B(7, 0, 1) C(-3, -2, -5) and…
  14. Using vectors, find the value of lambda such that the points (lambda ,-10 , 3)…
Exercise 23.8
  1. 2 i + j - k , 3 i-2 j + k and i+4 j-3 k Show that the points whose position…
  2. 3 i+2 j+4 k , i + j + k and - i+4 j-2 k Show that the points whose position…
  3. A(6, -7, -1), B(2 -3, 1) and C(4, -5, 0) Using vector method, prove that the…
  4. A(2, -1, 3), B(4, 3, 1) and C(3, 1, 2) Using vector method, prove that the…
  5. A(1, 2, 7), B(2, 6, 3) and C(3, 10 -1) Using vector method, prove that the…
  6. A(-3, -2, -5), B(1, 2, 3) and C(3, 4, 7) Using vector method, prove that the…
  7. A (2, -1, 3), B (3, -5, 1) and C (-1, 11, 9). Using vector method, prove that…
  8. If vector a , vector b , vector c are non-zero, non-coplanar vectors, prove…
  9. If vector a , vector b , vector c are non-zero, non-coplanar vectors, prove…
  10. Show that the four points having position vectors 6 i-7 j , 16 i-19 j-4 k , 3…
  11. 2 i - j + k , i - 3 j-5 k and 3 i-4 j-4 k Prove that the following vectors are…
  12. i + j + k , 2 i+3 j - k and - i-2 j+2 k Prove that the following vectors are…
  13. Prove that the following vectors are non-coplanar : 3 i + j - k , 2 i - j+7 k…
  14. Prove that the following vectors are non-coplanar : i+2 j+3 k , 2 i + j+3 k…
  15. If vector a , vector b , vector c are non-coplanar vectors, prove that the…
  16. If vector a , vector b , vector c are non-coplanar vectors, prove that the…
  17. Show that the vectors vector a , vector b , vector c given by vector a = i+2…
  18. Prove that a necessary and sufficient condition for three vectors vector a ,…
  19. Show that the four points A, B, C and D with position vectors vector a ,…
Exercise 23.9
  1. Can a vector have direction angles 45o, 60o, 120o.
  2. Prove that 1, 1, and 1 cannot be direction cosines of a straight line.…
  3. A vector makes an angle of π/4 with each of x - axis and y - axis. Find the…
  4. A vector vector i is inclined at equal acute angles to x - axis, y - axis, and…
  5. A vector vector i is inclined to the x - axis at 45o and y - axis at 60o. If |…
  6. Find the direction cosines of the following vectors : i. 2 i+2 j - k ii. 6 i-2…
  7. Find the angles at which the following vectors are inclined to each of the…
  8. Show that the vector i + mathfrakj + k is equally inclined with the axes OX, OY…
  9. Show that the direction cosines of a vector equally inclined to the axes OX, OY…
  10. If a unit vector vector a makes an angle pi /3 with i , pi /4 with j and an…
  11. Find a vector vector i of magnitude 3 root 2 units which makes an angle of pi…
  12. A vector vector i is inclined at equal angles to the three axes. If the…

Exercise 23.1
Question 1.

Represent the following graphically:

i. a displacement of 40 km, 30° east of north

ii. a displacement of 50 km south - east

iii. a displacement of 70 km, 40° north of west.


Answer:

i. a displacement of 40 km, 30° east of north


Step 1: Draw north, south, east and west as shown below:



Step 2: Plot a line 30° east of north as shown below:



Step 3: Define scale and mark 40km on line


Let the scale be 10km = 1cm



represents the displacement of 40 km, 30o East of North


ii. a displacement of 50 km south - east


Step 1: Draw north, south, east and west as shown below:



Step 2: As the displacement should be south - east, the angle between the displacement and east (or south) will be 45°. Now, plot a line 45° east of south as shown below:



Step 3: Define scale and mark point R such that OR = 50km on line . Let the scale be 10km = 1cm



represents the displacement of 50 km south – east


iii. A displacement of 70 km, 40o north of west.


Step 1: Draw north, south, east and west as shown below:



Step 2: Plot a line 40° north of west as shown below:



Step 3: Define scale and mark point R such that OR = 70km on line .Let the scale be 10km = 1cm



represents the displacement of 70 km, 40o north of west



Question 2.

Classify the following measures as scalars and vectors :

i. 15 kg

ii. 20 kg weight

iii. 45°

iv. 10 meters south - east

v. 50 m/sec2


Answer:

i. 15 kg - is a scalar quantity as this involves only mass. A scalar quantity is a one - dimensional measurement of a quantity, like temperature, or mass.


ii. 20 kg weight - is a vector quantity as it involves both magnitude and direction. Weight is a force which is a vector and has a magnitude and direction.


iii. 45o is a scalar quantity as it involves the only magnitude. A scalar quantity is a one - dimensional measurement of a quantity, like temperature, or mass.


iv. 10 meters south - east is a vector quantity as it involves both magnitude and direction.


v. 50 m/sec2 is a scalar quantity as it involves a magnitude of acceleration. A scalar quantity is a one - dimensional measurement of a quantity.



Question 3.

Classify the following as scalars and vector quantities:

i. Time period

ii. Distance

iii. Displacement

iv. Force

v. Work

vi. Velocity

vii. Acceleration


Answer:

i. Time period - is a scalar quantity as it involves only magnitude. A scalar quantity is a one - dimensional measurement of a quantity. Eg: 10 seconds has only magnitude, i.e., 10 and no direction.


ii. Distance - is a scalar quantity as it involves only magnitude. A scalar quantity is a one dimensional measurement of a quantity. Eg: 5meters has only magnitude 5 and no direction.


iii. Displacement - is vector quantity as it involves both magnitude and direction. Vector quantity has both magnitude and direction.


iv. Force - is a vector quantity as it involves both magnitude and direction. Vector quantity has both magnitude and direction. Eg., 5N downward has magnitude of 5 and direction is downward.


v. Work done - is a scalar quantity as it involves only magnitude and no particular direction. A scalar quantity is a one dimensional measurement of a quantity.


vi. Velocity - is a vector quantity as it involves both magnitude as well as direction. Vector quantity has both magnitude and direction. Eg., 5m/s east has magnitude of 5m/s and also direction towards east.


vii. Acceleration is a vector quantity because it involves both magnitude as well as direction.



Question 4.

In fig 23.5 ABCD is a regular hexagon, which vectors are:



i. Collinear

ii. Equal

iii. Cointitial

iv. Collinear but not equal.


Answer:

i. Collinear


Two or more vectors that lie on the same line or on a parallel line to this are called collinear vectors. Two collinear vectors may point in either same or opposite direction. But, they cannot be inclined at some angle from each other.


Hence FE (), AD () and BC () are collinear vectors.


And also AF () and CD () are collinear vectors.


And AB () and ED () are collinear vectors.


ii. Equal


Equal vectors are vectors that have the same magnitude and the same direction. Equal vectors may start at different positions.


Hence AF () and CD () are equal vectors.


And also FE () and BC () are equal vectors.


And AB () and ED () are equal vectors.


iii. Co - initial


Any given two vectors are called co - initial vectors if both the given vectors have the same initial point.


Hence, AB (), AF () and AD () are co - initial vectors.


iv. Collinear but not equal.


And AD () and BC () are collinear but not equal vectors.


And AD () and FE () are collinear but not equal vectors



Question 5.

Answer the following as true or false:

i. and are collinear.

ii. Two collinear vectors are always equal in magnitude.

iii. Zero vector is unique.

iv. Two vectors having same magnitude are collinear.

v. Two collinear vectors having the same magnitude are equal.


Answer:

i. and are collinear. (True)


Two or more vectors that lie on the same line or on a parallel line to this are called collinear vectors.


and are collinear.


ii. Two collinear vectors are always equal in magnitude. (False)


Two or more vectors that lie on the same line or on a parallel line to this are called collinear vectors. Two collinear vectors may point in either same or opposite direction. And they are not necessarily equal in magnitude they can be of different magnitude also.


iii. Zero vector is unique.(True)


There is only one zero - vector in a vector space. Hence zero vector is unique.


iv. Two vectors having same magnitude are collinear. (False)


It is not necessary for two vectors having the same magnitude to be parallel to the same line. Hence two vectors having same magnitude need not be collinear.


v. Two collinear vectors having the same magnitude are equal.(False)


Two vectors are said to be equal if they have the same magnitude and direction, regardless of the positions of their initial points.




Very Short Answer
Question 1.

Define “zero vector”.


Answer:

Zero vector is a vector which has magnitude is 0. It is denoted by .



Question 2.

Define unit vector.


Answer:

A unit vector is a vector whose magnitude is 1. It is denoted by capping the vector whose unit vector is required. For instance, the unit vector of will be .



Question 3.

Define position vector of a point.


Answer:

A position vector is a vector which tells the relative position of any point in space with respect to origin. This vector starts from origin and its head lies on the point itself. If The x, y, z coordinates of the point is x1, y1, z1, the position vector will be equal to .



Question 4.

Write in the simplified form.


Answer:

Any vector , if the position vectors of point P () and Q () are known, can be written as .

Let the position vectors of points P, Q, R be .


Then =



Question 5.

If are two non-collinear vectors such that , then write the values of x and y.


Answer:

Since and are non-colinear vectors, the only way the equality will hold is if x=y=0.



Question 6.

If represent two adjacent sides of a parallelogram, then write vectors representing its diagonals.


Answer:


In the above figure, , , and



Using parallelogram law of vector addition, we can say that


and or


Also, are the diagonals of the parallelogram.


Hence the diagonal vectors of a parallelogram formed by vectors and will be ) and .



Question 7.

If represent the sides of a triangle taken in order, then write the value of


Answer:

Let ∆ABC be the required triangle with and .

Any vector , if the position vectors of point A () and B () are known, can be written as .


Let the position vectors of points A, B, C be .


Then



Question 8.

If are position vectors of the vertices A, B and C respectively, of a triangle ABC, write the value of .


Answer:

Any vector , if the position vectors of point A () and B () are known, can be written as .

Since the position vectors of points A, B, C are , we get




Question 9.

If are the position vectors of the vertices of a triangle, then write the position vector of its centroid.


Answer:


In the figure, D is the mid-point of AB, so it divides AB in 1:1 ratio. CD is a median of ∆ABC. G is the centroid of the triangle and by the property of triangle, G divides CD in 2:1 ratio.


The position vector of point D can be calculated using the section formula for vector, which states that the position vector of a point() dividing two position vectors() in ration m:n, internally is



So,


Similarly, using section formula for G between points C and D, we get




Question 10.

If G denotes the centroid of Δ ABC, then write the value of


Answer:

Let the position vector points A, B, C be and . Then the position vector of G will be .

Any vector , if the position vectors of point A () and B () are known, can be written as .


Then,




Question 11.

If denote the position vectors of points A and B respectively and C is a point on AB such that 3AC = 2AB, then write the position vector of C.


Answer:

Given that the position vector points A, B are .

Let us assume that C lies between A and B.


Then AB=AC+BC


Given that 3AC=2AB=2(AC+BC)


⇒ AC=2BC


Therefore AC: BC=1:2


Also, since the ratio is positive, our assumption was correct.


Using section formula,



Question 12.

If D is the mid-point of side BC of a triangle ABC such that write the value of λ.


Answer:

Let the position vectors of A, B and C be and

Then the position vector of will be


Any vector , if the position vectors of point A () and B () are known, can be written as .


Then,



Substituting value of , we get



or λ=2



Question 13.

If D, E, F are the mid-points of the sides BC, CA and AB respectively of a triangle ABC, write the value of


Answer:

Let the position vectors of A, B and C be and

Then the position vector of and will be , and respectively.


Any vector , if the position vectors of point A () and B () are known, can be written as .


Then,





Question 14.

If is non-zero vector of modulus a and m is a non-zero scalar such that m is a unit vector, write the value of m.


Answer:

The modulus of is a, therefore can be written as modulus×unit-direction=

Given that m has the magnitude of 1, therefore has magnitude of 1 or ma=1.Hence



Question 15.

If are the position vectors of the vertices of an equilateral triangle whose orthocentre is at the origin, then write the value of .


Answer:

Since in an equilateral triangle, orthocenter and centroid coincide, therefore the position vector of centroid is .

Also, the position vector of centroid G () can be defined as


Therefore, hence



Question 16.

Write a unit vector making equal acute angles with a coordinates axes.


Answer:

Let the angle made be α. We know that the sum of squares of direction cosines of a vector is 1. SO, we get

cos2α+cos2α+cos2α=1



Since, α is acute therefore


Any vector, if it’s magnitude and direction cosines are given can be written as


So the required vector is


Rationalizing, we get



Question 17.

If a vector makes angles α, β, γ with OX, OY and OZ respectively, then write the value of sin2α + sin2β + sin2γ.


Answer:

The sum of squares of direction cosines of a vector is 1.

Let the angles made by vector be α, β, γ. Then, we get


cos2α+cos2β+cos2γ=1


using cos2θ=1-sin2θ, we get


(1-sin2α) +(1-sin2β) +(1-sin2γ) =1


Or, sin2α+sin2β+sin2γ=2



Question 18.

If are position vectors of the points A, B and C respectively, write the value of .


Answer:

Any vector , if the position vectors of point A () and B () are known, can be written as .

Since the position vectors of points A, B, C are , we get




Question 19.

Write a vector of magnitude 12 units which makes 45° angle with X-axis, 60° angle with y-axis and an obtuse angle with Z-axis.


Answer:

Let the angles made by vector be α, β, γ and the magnitude be m.

Given that α=45°, β=60° and m= 12. We have to figure out the vector.


Since cos2α+cos2β+cos2γ=1, we get


cos245°+cos260°+cos2γ=1



Since γ is obtuse, .


Any vector, if it’s magnitude and direction cosines are given can be written as


So the required vector is


Rationalizing, we get



Question 20.

Write the length (magnitude) of a vector whose projections on the coordinate axes are 12, 3 and 4 units.


Answer:

Since Lx=12, Ly=3 and Lz=4 are given, we can find out L by

L2= Lx2+Ly2+Lz2


=122+32+42


=144+9+16=169


Hence L=13 units.



Question 21.

Write the position vector of a point dividing the line segment joining points A and B with position vectors externally in the ratio 1 : 4, where and


Answer:

If a point R() divides the vector joining point P() and Q() externally in the ratio m:n, then


Here, , , m=1 and n=4


Then



Question 22.

Write the direction cosines of the vector


Answer:

The direction cosines of a vector are



In this question, a1=6, a2=-2 and a3=3, Substituting in formulas we get




Question 23.

If write unit vectors parallel to


Answer:

Given that , we get


The unit vector of any vector can be written as


Let


Then,


Both will be parallel to , therefore the answer is



Question 24.

If write a unit vector along the vector


Answer:

Given that , we get


The unit vector of any vector can be written as


Let


Then,


The unit vector in direction of is



Question 25.

Write the position vector of a point dividing the line segment joining points having position vectors externally in the ratio 2 : 3.


Answer:

. If a point R() divides the vector joining point P() and Q() externally in the ratio m:n, then


Here, , , m=2 and n=3


Then



Question 26.

If fine the unit vector in the direction of


Answer:

Given that , we get


The unit vector of any vector can be written as


Let


Then,



Question 27.

If and find


Answer:

Given that , we get


The magnitude of any vector can be written as




Question 28.

A unit vector makes angles respectively and a acute angle θ with Find θ.


Answer:

The sum of squares of direction cosines of a vector is 1.

Let the angles made by vector be α, β, θ. Then, we get


cos2α+cos2β+cos2θ=1


Given that , we have to calculate θ



(Since θ is acute)


Hence,



Question 29.

Write a unit vector in the direction of


Answer:

The unit vector of any vector can be written as

Given that


We get



Question 30.

If and find a unit vector parallel to


Answer:

Given that , we get


The unit vector of any vector can be written as


Let


Then,


Both will be parallel to , therefore the answer is



Question 31.

Write a unit vector in the direction of


Answer:

The unit vector of any vector can be written as

Given that


We get



Question 32.

Find the position vector of the mid-point of the line segment AB, where A is the point (3, 4, –2) and B is the point (1, 2, 4).


Answer:

If the co-ordinates of a point A≡ (x1, y1, z1), then the position vector of A() is


Given that A≡(3,4,2) and B≡(1,2,4), we get position vector of A() and B(). Let the midpoint be C()



The position vector of midpoint of two vectors is defined by




Question 33.

Find a vector in the direction of which has magnitude of 6 units.


Answer:

Given that

The unit vector of any vector can be written as



Let the required vector be .


Any vector () with magnitude m and unit vector can be written as .


Since the magnitude of is 6 and it’s unit vector is , we get




Question 34.

What is the cosine of the angle which the vector makes with y-axis?


Answer:

The angle that a vector makes with y-axis is


In this question, a1=, a2=1 and a3=1, Substituting in formulas we get




Question 35.

Write two different vectors having same magnitude.


Answer:

and both have magnitude 1 but different directions. is along x-axis and is along y-axis.



Question 36.

Write two different vectors having same direction.


Answer:

and both have the same direction but different magnitudes, 1 and 2.



Question 37.

Write a vector in the direction of vector which has magnitude of 8 unit.


Answer:

Given that

The unit vector of any vector can be written as



Let the required vector be .


Any vector () with magnitude m and unit vector can be written as .


Since the magnitude of is 8 and it’s unit vector is , we get




Question 38.

Write the direction cosines of the vector


Answer:

The direction cosines of a vector are



In this question, a1=1, a2=2 and a3=3, Substituting in formulas we get




Question 39.

Find a unit vector in the direction of


Answer:

Given that

The unit vector of any vector can be written as




Question 40.

For what value of ‘a’ the vectors and are collinear?


Answer:

Two vectors and are collinear , if and only if, .

Here and


Hence,


Solving this equality, we get


a=-4



Question 41.

Write the direction cosines of the vectors


Answer:

The direction cosines of a vector are



In this question, a1=-2, a2=1 and a3=-5, Substituting in formulas we get




Question 42.

Find the sum of the following vectors:


Answer:


Then



Question 43.

Find a unit vector in the direction of the vector


Answer:

Given that

The unit vector of any vector can be written as




Question 44.

If and are two equal vectors, then write the value of x + y + z.


Answer:

Two vectors and are equal , if and only if, a1=b1, a2=b2, a3=b3

Here and


Hence, we get x=3, -y=2⇒y=-2 and -z=1⇒ z=-1



Question 45.

Write a unit vector in the direction of the sum of the vectors and


Answer:

and


Let


The unit vector of any vector can be written as


Hence,



Question 46.

Find the value of ‘p’ for which the vectors and are parallel.


Answer:

Two vectors and are parallel , if and only if, .

Here and


Hence,


Solving this equality, we get




Question 47.

Find a vector of magnitude making an angle of π/4 with x-axis π/2 with y-axis and an acute angle θ with z-axis.


Answer:

Let the angles made by vector be α, β, θ and the magnitude be m.

Given that . We have to figure out the vector.


Since cos2α+cos2β+cos2θ=1, we get




Since θ is acute, .


Any vector, if it’s magnitude and direction cosines are given can be written as


So, the required vector is



Question 48.

Write a unit vector in the direction of , where P and Q are the points (1, 3, 0) and (4, 5, 6) respectively.


Answer:

If the co-ordinates of points A≡ (x1, y1, z1) and B≡ (x2, y2, z2), then the vector is


Given that P≡(1,3,0) and Q≡(4,5,6), we get



The unit vector of any vector can be written as


Hence,



Question 49.

Find a vector in the direction of vector which has magnitude 21 units.


Answer:

Given that

The unit vector of any vector can be written as



Let the required vector be .


Any vector () with magnitude m and unit vector can be written as .


Since the magnitude of is 21 and it’s unit vector is , we get




Question 50.

If and –3 ≤ λ ≤ 2, then write the range of


Answer:

Given that and -3≤λ≤2

We have to figure out range of


In calculating the modulus of a vector multiplied by a scalar quantity, the sign of the scalar quantity does not matter, only it’s absolute value does.


Hence the minimum value of when λ=0 and maximum value of when λ=-3.



Question 51.

In a triangle OAC, if B is the mid-point of side AC and then what is ?


Answer:

Given that the position vectors of point A and B are and . Let the position vector of point C be .

The position vector of B will be defined as





Question 52.

Write the position vector of the point which divides the join of points with position vectors and in the ratio 2:1.


Answer:

If a point R() divides the vector joining point P() and Q() externally in the ratio m:n, then


Here, , , m=2 and n=1


We get,




Mcq
Question 1.

Mark the correct alternative in each of the following:

If in a Δ ABC, A ≡ (0, 0), B ≡ (3, 3, √3), C ≡ (–3, √3, 3), then the vector of magnitude units directed along AO, where O is the circumcentre of Δ ABC is

A.

B.

C.

D. none of these


Answer:


Slope of a line joining two points


Slope of AC




Slope of AB




Product of Slopes (AC AB) =


= -1


As the Product of Slopes (AC AB) = -1, so AC ⟘ AB, ie.., CAB = 90°.


Circumcentre (O) of Triangle ABC = Mid-Point of BC


Mid-Point of BC =




Now,





Unit Vector



Vector along , whose magnitude is




Option (A) is the answer.


Question 2.

Mark the correct alternative in each of the following:

If are the vectors forming consecutive sides of a regular hexagon ABCDEF, then the vector representing side CD is

A.

B.

C.

D.


Answer:


--------- (i)


As, AD = 2BC {Properties of a regular hexagon, also AD || BC (Parallel)}


Putting in equation (i),







Option(C)is the answer.


Question 3.

Mark the correct alternative in each of the following:

Forces act along OA and OB. If their resultant passes through C on AB, then

A. C is a mid-point of AB

B. C divides AB in the ratio 2 : 1

C. 3 AC = 5 CB

D. 2AC = 3 CB


Answer:


Draw ON perpendicular to the line AB.


Let be the unit vector along ON,


The resultant force --------- (i)


The angles between and the forces , , are CON, AON & BON respectively,







R = 8


---------- (i)


Multiplying the equation (i) by 3,


----------(ii)


Also, ----------(iv)


Multiplying the equation (iv) by 5,


----------(v)


Adding equation (iv) & (v) respectively,






3AC = 5CB


Option(C)is the answer.


Question 4.

Mark the correct alternative in each of the following:

If are three non-zero vectors, no two of which are collinear and the vector is collinear with is collinear with , then

A.

B.

C.

D. none of these


Answer:

As is collinear with ,


-------- (i)


As is collinear with ,


-------- (ii)


Adding both sides of the equation (i),



-------- (iii)


Adding both sides of the equation (iii),



--------- (iv)


Equating the RHS of equation (iii)& (iv), being their LHS equal,



As, a is not collinear with c,



{From equation (iv)}



Option (D)is the answer.


Question 5.

Mark the correct alternative in each of the following:

If and points are collinear, then a is equal to

A. 40

B. –40

C. 20

D. –20


Answer:

A (, B () & C () are collinear,


Then











Comparing the LHS & RHS of the above mentioned equation,






-80 = a – 40


a = 40 – 80


a = -40


a = -40


Option (B) is the answer.


Question 6.

Mark the correct alternative in each of the following:

If G is the intersection of diagonals of a parallelogram ABCD and O is any point, then

A.

B.

C.

D.


Answer:


Let us consider the point O as origin.


G is the mid – point of AC.



------- (i)


Also, G is the mid− point of BD,



------- (ii)


Adding eq. (i) & eq. (ii),





Option (B) is the answer


Question 7.

Mark the correct alternative in each of the following:

The vector is a

A. null vector

B. unit vector

C. constant vector

D. none of these


Answer:








=1


Hence, the given vector is a unit vector.


Option (B) is the answer


Question 8.

Mark the correct alternative in each of the following:

In a regular hexagon ABCDEF, Then,

A.

B.

C.

D.


Answer:



------- (i)


In the triangle ADE,



{}



Option (C) is the answer


Question 9.

Mark the correct alternative in each of the following:

The vector equation of the plane passing through is provided that

A. α + β + γ = 0

B.α + β + γ = 1

C. α + β = γ

D. α2 + β2 + γ2 = 1


Answer:

As a plane passing through


Lines and lie on the plane.


The parametric equation of the plane can be expressed as,




As,





Option (B) is the answer.


Question 10.

Mark the correct alternative in each of the following:

If O and O’ are circumcentre and orthocentre of Δ ABC, then equals

A.

B.

C.

D.


Answer:


Let the vertices of the triangle ABC be A (), B () & C (), with respect to the origin.


O (x,y) is the circumcentre & O’ (0,0) is the orthocenter.



As the centroid ‘G’ divides the orthocentre ‘C’ (x,y) and circumcentre (0,0) in the ratio 2 : 1.


By using Section Formula,







Option (B)is the answer.


Question 11.

Mark the correct alternative in each of the following:

If are the position vectors of points A, B, C, D such that no three of them are collinear and then ABCD is a

A. rhombus

B. rectangle

C. square

D. parallelogram


Answer:




And,




(Given)


Multiplying the aqbove mentioned equation by ,



So, the position vector of mid – point of BD = Position Vector of mid - point of AC.


Hence the diagonals bisect each other.


Therefore the given figure ABCD is a parallelogram.


Option (D) is the answer


Question 12.

Mark the correct alternative in each of the following:

Let G be the centroid of Δ ABC. If then the bisector in terms of is

A.

B.

C.

D.


Answer:


Let A be the origin, then , implies that the position vectors of B and C are & respectively.


Let AD be the median and G be the centroid.


Then,


Position Vector of


Position Vector of




Option (A) is the answer.


Question 13.

Mark the correct alternative in each of the following:

If ABCDEF is a regular hexagon, then equals.

A.

B.

C.

D.


Answer:








As,


In triangle AOF,





And,






Option (D) is the answer


Question 14.

Mark the correct alternative in each of the following:

The position vectors of the points A, B, C are and respectively. These points

A. form an isosceles triangle

B. form a right triangle

C. are collinear

D. form a scalene triangle


Answer:





















Hence the triangle is isosceles with two sides equal.


Option (A) is the answer


Question 15.

Mark the correct alternative in each of the following:

If three points A, B and C have position vectors and respectively are collinear, then (x, y) =

A. (2, –3)

B. (–2, 3)

C. (–2, –3)

D. (2, 3)


Answer:








the given vectors are collinear.




After comparing the equations,





y – 3 = -6


y = -6 + 3


y = -3



4 – x = 2


x = 4 – 2


x = 2


(x, y) = (2, -3)


Option (A) is the answer.


Question 16.

Mark the correct alternative in each of the following:

ABCD is a parallelogram with AC and BD as diagonals. Then,

A.

B.

C.

D.


Answer:

ABCD is a parallelogram with diagonals AC and BD.


&





{}




Option (C) is the answer.


Question 17.

Mark the correct alternative in each of the following:

If OACB is a parallelogram with and , then

A.

B.

C.

D.


Answer:

,




{}


-------(i)






Option (D) is the answer.


Question 18.

Mark the correct alternative in each of the following:

If are two collinear vectors, then which of the following are incorrect?

A. for some scalar λ

B.

C. the respective components of are proportional

D. both the vectors have the same direction but different magnitudes


Answer:

If & are collinear vectors, then they are parallel,


Then,


For some scalar


If , then,



If









Thus, respective components of are proportional.


However the vectors can have different directions.


Statement given in D is incorrect.


Option (D) is the answer.


Question 19.

Mark the correct alternative in each of the following:

If figure which of the following is not true?



A.

B.

C.

D.


Answer:


Subtracting, from both the sides of the above mentioned equation,



Solving RHS,




LHS ≠ RHS


Hence, it is not true.



Option (C) is the answer.



Exercise 23.2
Question 1.

If P, Q and R are three collinear points such that and Find the vector .


Answer:


As P, Q and R are three collinear points.


Hence, as shown in above fig


And given and .


Therefore



Question 2.

Give a condition that three vectors and form the three sides of a triangle. What are the other possibilities?


Answer:

Given that, and are three sides of a triangle.



Hence from the above figure we get,


AB = , BC = and AC =


So


[since]


[Since CA = - AC]


Triangle law says that, if vectors are represented in magnitude and direction by the two sides of a triangle is same order, then their sum is represented by the third side took in reverse order. Thus,


or or



Question 3.

If and are two non-collinear vectors having the same initial point. What are the vectors represented by and


Answer:

Given and are two non - collinear vectors having the same initial point.


Let and


Let us draw a parallelogram with AB and AD as any of the two sides of the parallelogram as shown below.



We know in parallelogram opposite sides are equal hence,


and


Now consider ΔABC, applying triangles law of vectors, we get



Similarly in ΔABD, applying triangles law of vectors, we get



Looking at the two equations (i) and (ii) we can conclude that


andare the diagonals of a parallelogram whose adjacent sides areand.



Question 4.

If is a vector and m is a scalar such that m then what are the alternatives for m and ?


Answer:

Given is a vector and m is a scalar such that


Let then according to the given question





Compare the coefficients of, we get


ma1 = 0 ⇒ m = 0 or a1 = 0


Similarly, mb1 = 0 ⇒ m = 0 or b1 = 0


And, mc1 = 0 ⇒ m = 0 or c1 = 0


From the above three conditions,


m = 0 or a1 = b1 = c1 = 0


⇒ m = 0 or


Hence the alternatives for m and are m = 0 or



Question 5.

If are two vectors, then write the truth value of the following statements :



Answer:

Given:


Let and


So according to the given criteria,





Compare the coefficients of, we get


a1 = a2, b1 = b2 and c1 = c2……….(i)


and


Substitute the values from eqn (i) in above eqn we get




But,


Hence


Therefore,



Question 6.

If are two vectors, then write the truth value of the following statements :



Answer:

Given:


It means the magnitude of the vector is equal to the magnitude of the vector, but we cannot conclude anything about the direction of the vector.


So it is false that



Question 7.

If are two vectors, then write the truth value of the following statements :



Answer:

Given:

It means the magnitude of the vector is equal to the magnitude of the vector, but we cannot conclude anything about the direction of the vector.


And we know that means magnitude and same direction. So, it is false that



Question 8.

ABCD is a quadrilateral. Find the sum of the vectors and


Answer:

Given: ABCD is a quadrilateral as shown below



Consider ΔADC and apply triangle law of vector, we get



Similarly, consider ΔABC and apply triangle law of vector, we get



Substituting the value of from eqn(i) into eqn(ii), we get



Now add on both sides, we get




Hence sum of the vectors and is



Question 9.

ABCDE is a pentagon, prove that



Answer:

Given: ABCDE is a pentagon as shown below



Consider ΔABC and apply triangle law of vector, we get



Similarly, consider ΔACD and apply triangle law of vector, we get



And, consider ΔADE and apply triangle law of vector, we get



Adding (i), (ii) and (iii), we get





[as



Hence proved



Question 10.

ABCDE is a pentagon, prove that



Answer:

Given: ABCDE is a pentagon as shown below


Consider ΔABC and apply triangle law of vector, we get



Similarly, consider ΔADE and apply triangle law of vector, we get



And, consider ΔADC and apply triangle law of vector, we get



Adding (i), (ii) and (iii), we get




Add on both sides we get,




Or


Hence proved.



Question 11.

Prove that the sum of all vectors drawn from the centre of a regular octagon to its vertices is the zero vector.


Answer:

Given: a regular octagon


To prove the sum of all vectors drawn from the centre of a regular octagon to its vertices is the zero vector


Proof:


Let O be the centre of a regular octagon, we know that the centre of a regular octagon bisects all the diagonals passing through it as shown in figure below







Thus,



The sum of all vectors drawn from the centre of a regular octagon to its vertices is



Substitute the values from eqn(i) in above eqn, we get





Hence, the sum of all vectors drawn from the centre of a regular octagon to its vertices is a zero vector.


Hence, proved.



Question 12.

If P is a point and ABCD is a quadrilateral and show that ABCD is a parallelogram.


Answer:

Given a quadrilateral ABCD, P is a point outside the quadrilateral and



[given]


Or,


………..(i)[as ]


Consider ΔAPB and apply triangle law of vector, we get



And consider ΔDPC and apply triangle law of vector, we get



Substitute the values from eqn(ii) an eqn(iii) in eqn(i), we get



Therefore, AB is parallel to DC and equal is magnitude.


Hence, ABCD is a parallelogram.


Hence proved



Question 13.

Five forces and and act at the vertex of a regular hexagon ABCDEF. Prove that the resultant is 6 where O is the centre of hexagon.


Answer:








Given a regular hexagon ABCDEF with O as the centre of the hexagon as shown in figure below


To prove


We know that centre O of the hexagon bisects the diagonals


∴, ,


Consider ΔABO and apply triangle law of vector, we get



And consider ΔACO and apply triangle law of vector, we get



And consider ΔAEO and apply triangle law of vector, we get



And consider ΔAFO and apply triangle law of vector, we get



Now,



Substitute the corresponding values from eqn(i) to eqn(v) in above eqn, we get



[from eqn(i)]


[as and ]




Hence


Therefore the resultant of the five forces and is


Hence proved




Exercise 23.3
Question 1.

Find the position vector of a point R which divides the line joining the two points P and Q with position vectors and respectively in the ratio 1 : 2 internally and externally.


Answer:

Let the position vectors of points P, Q and R be, and respectively.


Given and


(i) R divides PQ internally in the ratio 1:2



Recall the position vector of point P which divides AB, the line joining points A and B with position vectors and respectively, internally in the ratio m : n is



Here, m = 1 and n = 2.




We have and





Thus, the position vector of point R is .


(ii) R divides PQ externally in the ratio 1:2



Recall the position vector of point P which divides AB, the line joining points A and B with position vectors and respectively, externally in the ratio m : n is



Here, m = 1 and n = 2.





We have and





Thus, the position vector of point R is .



Question 2.

If are the position vectors of A, B respectively, find the position vector of a point C in AB produced such that AC = 3 AB and that a point D in BA produced such that BD = 2 BA.


Answer:

Given the position vectors of points A and B are and.


Let the position vectors of points C and D be and.



We have AC = 3AB.


From the above figure, observe AB = AC – BC


⇒ AC = 3 (AC – BC)


⇒ AC = 3AC – 3BC


⇒ 2AC = 3BC


∴ AC : BC = 3 : 2


So, C divides AB externally in the ratio 3:2.


Recall the position vector of point P which divides AB, the line joining points A and B with position vectors and respectively, externally in the ratio m : n is



Here, m = 3 and n = 2


So, the position vector of C is





We also have BD = 2BA.


From the figure, observe BA = BD – AD


⇒ BD = 2 (BD – AD)


⇒ BD = 2BD – 2AD


⇒ BD = 2AD


∴ BD : AD = 2 : 1


So, D divides BA externally in the ratio 2:1.


We now use the same formula as earlier to find the position vector of D.


Here, m = 2 and n = 1





Thus, the position vector of point C is and the position vector of point D is .



Question 3.

Show that the four points A, B, C, D with position vectors respectively such that are coplanar. Also, find the position vector of the point of intersection of the line segments AC and BD.


Answer:

Given the position vectors of points A, B, C and D are,, and respectively.


We have


Rearranging the terms in the above equation,



Observe that the sum of coefficients on the LHS of this equation (3 + 5 = 8) is equal to that on the RHS (2 + 6 = 8).


We now divide the equation with 8 on both sides.




Now, consider the LHS of this equation.


Let , the position vector of some point X.



Recall the position vector of point P which divides AB, the line joining points A and B with position vectors and respectively, internally in the ratio m : n is



Here, m = 3 and n = 5


So, X divides CA internally in the ratio 3:5.


Similarly, considering the RHS of this equation, we have the same point X dividing DB in the ratio 2:6.


So, the point X lies on both the line segments AC and BD making it the point of intersection of AC and BD.


As AC and BD are two straight lines having a common point, we have all the points A, B, C and D lying in the same plane.


Thus, the points A, B, C and D are coplanar and in addition, the position vector of the point of intersection of line segments AC and BD is or.



Question 4.

Show that the four points P, Q, R, S with position vectors respectively such that are coplanar. Also, find the position vector of the point of intersection of the line segments PR and QS.


Answer:

Given the position vectors of points P, Q, R and S are,, and respectively.


We have


Rearranging the terms in the above equation,



Observe that the sum of coefficients on the LHS of this equation (5 + 6 = 11) is equal to that on the RHS (2 + 9 = 11).


We now divide the equation with 11 on both sides.




Now, consider the LHS of this equation.


Let , the position vector of some point X.



Recall the position vector of point P which divides AB, the line joining points A and B with position vectors and respectively, internally in the ratio m : n is



Here, m = 5 and n = 6


So, X divides RP internally in the ratio 5:6.


Similarly, considering the RHS of this equation, we have the same point X dividing SQ in the ratio 2:9.


So, the point X lies on both the line segments PR and QS making it the point of intersection of PR and QS.


As PR and QS are two straight lines having a common point, we have all the points P, Q, R and S lying in the same plane.


Thus, the points P, Q, R and S are coplanar and in addition, the position vector of the point of intersection of line segments PR and QS is or.



Question 5.

The vertices A, B, C of triangle ABC have respectively position vectors with respect to a given origin O. Show that the point D where the bisector of ∠A meets BC has position vector where and


Answer:

Given the position vectors of vertices A, B and C of ΔABC are and respectively.


D is point on BC with position vector such that AD is the bisector of ∠A. I is the incenter of ΔABC.



Observe from the figure that D divides BC in the ratio BD:DC.


Using the angular bisector theorem, we know that the angle bisector of an angle in a triangle bisects the opposite side in the ratio equal to the ratio of the other two sides.



But, and.



Recall the vector is given by




Similarly,


So, we have .


Recall the position vector of point P which divides AB, the line joining points A and B with position vectors and respectively, internally in the ratio m : n is



Here, we have D dividing BC internally in the ratio m:n where m = BD = and n = DC =




Suppose and.



From angular bisector theorem above, we have.


Adding 1 to both sides,






In addition, as CI is the angular bisector of ∠C in ΔACD, using the angular bisector theorem, we have



So, we get


We have and



Assume



So, I divides AD in the ratio (β + γ):α.


Let the position vector of I be .


Using the aforementioned section formula, we can write



But, we already found.





Thus, and the position vector of the incenter is , where, and .



Question 6.

Let be the position vectors of the four distinct points A, B, C, D. If then show that ABCD is a parallelogram.


Answer:

Given the position vectors of points A, B, C and D are,, and respectively.



Recall the vector is given by




Similarly, the vector is given by




But, it is given that.



Two vectors are equal only when both their magnitudes and directions are equal.


and.


This means that the opposite sides in quadrilateral ABCD are parallel and equal.


Thus, ABCD is a parallelogram.




Exercise 23.4
Question 1.

If O is a point in space, ABC is a triangle and D, E, F are the mid-points of the sides BC, CA and AB respectively of the triangle, prove that


Answer:

Let position vectors of the vertices A, B and C of ΔABC with respect to O be , and respectively.



Let us also assume the position vectors of the midpoints D, E and F with respect to O are, and respectively.




Now, D is the midpoint of side BC.


This means D divides BC in the ratio 1:1.


Recall the position vector of point P which divides AB, the line joining points A and B with position vectors and respectively, internally in the ratio m : n is



Here, m = n = 1





Similarly, for midpoint E and side CA, we get and for midpoint F and side AB, we get .


Adding these three equations, we get






Thus, .



Question 2.

Show that the sum of the three vectors determined by the medians of a triangle directed from the vertices is zero.


Answer:

Consider a ΔABC with D, E and F being the midpoints of sides BC, CA and AB respectively.


Let the position vectors of these vertices and midpoints be as shown in the figure.


We need to prove.



As D is the midpoint of BC, using midpoint formula, we have



Similarly, and.


Recall the vector is given by




Similarly, and


Now, consider the vector.



But, and







Thus, the sum of the three vectors determined by the medians of a triangle is zero.



Question 3.

ABCD is a parallelogram and P is the point of intersection of its diagonals. If O is the origin of reference, show that


Answer:

Let position vectors of the vertices A, B, C and D of the parallelogram ABCD with respect to O be , , and respectively.



Also, let us assume position vector of P is .



Given ABCD is a parallelogram.


We know that the two diagonals of a parallelogram bisect each other. So, P is the midpoint of AC and BD.


As P is the midpoint of AC, using midpoint formula, we have





P is also the midpoint of BC.


So,


Now we have and.


Adding these two equations, we get




Thus.



Question 4.

Show that the line segments joining the midpoints of opposite sides of a quadrilateral bisect each other.


Answer:

Let ABCD be a quadrilateral. E, F, G and H are the midpoints of sides AB, BC, CD and DA respectively.


We need to prove EG and HF bisect each other. It is sufficient to show EFGH is a parallelogram, as the diagonals in a parallelogram bisect each other.


Let the position vectors of these vertices and midpoints be as shown in the figure.



As E is the midpoint of AB, using midpoint formula, we have



Similarly, , and .


Recall the vector is given by







Similarly






So, we have .


Two vectors are equal only when both their magnitudes and directions are equal.


and.


This means that the opposite sides in quadrilateral EFGH are parallel and equal, making EFGH a parallelogram.


EG and HF are diagonals of parallelogram EFGH. So, EG and HF bisect each other.


Thus, the line segments joining the midpoints of opposite sides of a quadrilateral bisect each other.



Question 5.

ABCD are four points in a plane and Q is the point of intersection of the lines joining the mid-points of AB and CD ; BC and AD. Show that where P is any point.


Answer:

Let E, F, G and H be the midpoints of sides AB, BC, CD and DA respectively of quadrilateral ABCD.


Let the position vectors of these vertices and midpoints be as shown in the figure.



As E is the midpoint of AB, using midpoint formula, we have



Similarly, , and .


We know that the line segments joining the midpoints of opposite sides of a quadrilateral bisect each other.


⇒ Q is the midpoint of EG and HF.


Once again using midpoint formula, we get


But, we found and .





Now, consider the vector .


Let the position vector of point P be .


Recall the vector is given by




Similarly, , and .





But, we found




Observe,




Thus,



Question 6.

Prove by vector method that the internal bisectors of the angles of a triangle are concurrent.


Answer:

Consider ΔABC with vertices A, B, C and sides BC = α, AC = β and AB = γ.


Let the position vectors of A, B and C be and respectively.


Let D and E (with position vectors and) be points on BC and AB such that AD and CE are the bisectors of ∠A and ∠C. Let, AB and CE meet at point I.



Observe from the figure that D divides BC in the ratio BD:DC.


Using the angular bisector theorem, we know that the angle bisector of an angle in a triangle bisects the opposite side in the ratio equal to the ratio of the other two sides.


(from our initial assumption)


Recall the position vector of point P which divides AB, the line joining points A and B with position vectors and respectively, internally in the ratio m : n is



Here, we have D dividing BC internally in the ratio m:n where m = γ and n = β.



From angular bisector theorem above, we had.


Adding 1 to both sides,






In addition, as CI is the angular bisector of ∠C in ΔACD, using the angular bisector theorem, we have



So, we get


So, I divides AD in the ratio (β + γ):α.


Let the position vector of I be .


Using the aforementioned section formula, we can write



But, we already found.





Now, observe E divides AB in the ratio AE:EB.


(from angular bisector theorem)


So, (using section formula)


By doing similar calculations as above for ∠C, we get



So, I divides CE in the ratio (α + β):γ.


Let the position vector of I now be .


Using the aforementioned section formula, we can write



But, we already found.





Observe that meaning the point I with position vector lies on both AB and CE.


Similarly, it can be shown that this point I also lies on the third angular bisector.


Thus, the internal bisectors of the angles of a triangle are concurrent with the point of concurrency given by the position vector where α,β and γ are sides of the ΔABC opposite to the vertices A, B and C respectively.




Exercise 23.5
Question 1.

If the position vector of a point (–4, –3) be find


Answer:

Given is the position vector of point (–4, –3).


We know position vector of a point (x, y) is given by, where and are unit vectors in X and Y directions.



Now, we need to find magnitude of i.e. .


Recall the magnitude of the vector is given as



Here, x = –4 and y = –3






Thus, .



Question 2.

If the position vector of a point (12, n) is such that find the value(s) of n.


Answer:

Given is the position vector of point (12, n).


We know position vector of a point (x, y) is given by, where and are unit vectors in X and Y directions.



Now, we need to find n such that.


Recall the magnitude of the vector is given as



Here, x = 12 and y = n




Squaring both the sides, we have







Thus, n = 5 or –5.



Question 3.

Find a vector of magnitude 4 units which is parallel to the vector


Answer:

Let be the required vector that is parallel to.


We know any vector parallel to a given vector is of the form, where λ is a real number.



Now, we need to find λ such that.


Recall the magnitude of the vector is given as



Here, x = and y = λ





Squaring both the sides, we have






Thus, the required vector is.



Question 4.

Express in terms of unit vectors and when the points are :

(i) A (4, -1), B(1, 3)

(ii) A(-6, 3), B(-2, -5)

Find in each case.


Answer:

(i) Given A = (4, –1) and B = (1, 3)


We know position vector of a point (x, y) is given by, where and are unit vectors in X and Y directions.


Let position vectors of points A and B be and respectively.



We also have.


Recall the vector is given by







Recall the magnitude of the vector is given as



Here, x = –3 and y = 4






Thus, and.


(ii) Given A = (–6, 3) and B = (–2, –5)


We know position vector of a point (x, y) is given by, where and are unit vectors in X and Y directions.


Let position vectors of points A and B be and respectively.



We also have.


Recall the vector is given by







Recall the magnitude of the vector is given as



Here, x = 4 and y = –8






Thus, and.



Question 5.

Find the coordinates of the tip of the position vector which is equivalent to where the coordinates of A and B are (-1, 3) and (-2, 1) respectively.


Answer:

Given A = (–1, 3) and B = (–2, 1)


We know position vector of a point (x, y) is given by, where and are unit vectors in X and Y directions.


Let position vectors of points A and B be and respectively.



We also have.


Recall the vector is given by







Now, it is given that there exists a point say (x, y) whose position vector is same as.


We know position vector of a point (x, y) is given by.




By comparing both the sides, we get x = –1 and y = –2


Thus, (–1, –2) is the tip of position vector that is same as .



Question 6.

ABCD is a parallelogram. If the coordinates of A, B and C are (–2, 1), (3, 0) and (1, –2), find the coordinates of D.


Answer:

Given A = (–2, –1), B = (3, 0) and C = (1, –2)


Let the other vertex D = (x, y)


We know position vector of a point (x, y) is given by, where and are unit vectors in X and Y directions.



Let position vectors of points A, B, C and D be,, and respectively.



We also have.


Similarly and.


Recall the vector is given by







Similarly, the vector is given by







But, it is given that ABCD is a parallelogram.


(as the opposite sides are parallel and equal)



By comparing both sides, we get 1 – x = 5 and 2 + y = –1


⇒ x = 1 – 5 = –4


and y = –1 – 2 = –3


So, x = –4 and y = –3


Thus, vertex D of parallelogram ABCD = (–4, –3).



Question 7.

If the position vectors of the points A (3, 4), B (5, -6) and C (4, -1) are respectively, compute


Answer:

Given A = (3, 4), B = (5, –6) and C = (4, –1)


We know position vector of a point (x, y) is given by, where and are unit vectors in X and Y directions.


(position vector of point A)


We also have


Similarly.


We need to compute.






Thus, .



Question 8.

If be the position vector whose tip is (5, -3), find the coordinates of a point B such that the coordinates of A being (4, -1).


Answer:

We know position vector of a point (x, y) is given by, where and are unit vectors in X and Y directions.


So, position vector of (5, –3) is



Given A = (4, –1) and let the coordinates of B = (x, y)


Let position vectors of points A and B be and respectively.



We also have.


Recall the vector is given by






But, it is given that



By comparing both sides, we get x – 4 = 5 and y + 1 = –3


⇒ x = 5 + 4 = 9


and y = –3 – 1 = –4


So, x = 9 and y = –4


Thus, coordinates of point B are (9, –4).



Question 9.

Show that the points and form an isosceles triangle.


Answer:

Let, and be the position vectors corresponding to the vertices A, B and C of ΔABC.



Recall the vector is given by







Recall the magnitude of the vector is given as



Now, we find the magnitude of .






Similarly, the vector is given by







Now, we find the magnitude of .





Similarly, the vector is given by







Now, we find the magnitude of .






Observe that which means the sides AB and AC of ΔABC are equal in length, making it an isosceles triangle.


Thus, the triangle formed by the given points is isosceles.



Question 10.

Find a unit vector parallel to the vector


Answer:

Let be the required vector that is parallel to.


We know any vector parallel to a given vector is of the form, where λ is a real number.



Now, we need to find λ such that.


Recall the magnitude of the vector is given as



Here, x = λ and y =





Squaring both the sides, we have






Thus, the required vector is.



Question 11.

The position vectors of points A, B and C are and respectively. If C divides the lien segment joining A and B in the ratio 3 : 1, find the values of λ and μ.


Answer:

Let the position vectors of points A, B and C be, and respectively.


Given:, and


C divides AB internally in the ratio 3:1.



Recall the position vector of point P which divides AB, the line joining points A and B with position vectors and respectively, internally in the ratio m : n is



Here, m = 3 and n = 1.




We have, and






By comparing both sides, we get 36 + λ = –44


⇒ λ = –44 – 36


∴ λ = –80


We also have 3μ + 3 = –12


⇒ 3μ = –15


∴ μ = –5


Thus, λ = –80 and μ = –5



Question 12.

Find the components along the coordinate axes of the position vector of each of the following points –

i. P (3, 2)

ii. Q (5, 1)

iii. R (–11, –9)

iv. S (4, –3)


Answer:

(i) Given P = (3, 2)


We know position vector of a point (x, y) is given by, where and are unit vectors in X and Y directions.


Let position vector of point P be.



So, component of along the X-axis is , that is a vector of magnitude 3 along the positive direction of the X-axis.


Also, component of along the Y-axis is , that is a vector of magnitude 2 along the positive direction of the Y-axis.


(ii) Given Q = (5, 1)


We know position vector of a point (x, y) is given by, where and are unit vectors in X and Y directions.


Let position vector of point Q be.



So, component of along the X-axis is 5, that is a vector of magnitude 5 along the positive direction of the X-axis.


Also, component of along the Y-axis is , that is a vector of magnitude 1 along the positive direction of the Y-axis.


(iii) Given R = (–11, –9)


We know position vector of a point (x, y) is given by, where and are unit vectors in X and Y directions.


Let position vector of point R be.



So, component of along the X-axis is , that is a vector of magnitude 11 along the negative direction of the X-axis.


Also, component of along the Y-axis is , that is a vector of magnitude 9 along the negative direction of the Y-axis.


(iv) Given S = (4, –3)


We know position vector of a point (x, y) is given by, where and are unit vectors in X and Y directions.


Let position vector of point S be.



So, component of along the X-axis is , that is a vector of magnitude 4 along the positive direction of the X-axis.


Also, component of along the Y-axis is , that is a vector of magnitude 3 along the negative direction of the Y-axis.




Exercise 23.6
Question 1.

Find the magnitude of the vector


Answer:

If a vector is given by then the magnitude of vector is generally denoted by which is equal to

So the magnitude






So the magnitude of the vector is 7.



Question 2.

Find the unit vector in the direction of


Answer:

Let the unit vector in the direction of


So any unit vector in the direction of



So the magnitude of the vector





So, the unit vector



Question 3.

Find a unit vector in the direction of the resultant of the vectors and


Answer:

To find the resultant vector we add all the vector by vector addition.




So, resultant vector is





So, the unit vector


Magnitude of






Question 4.

The adjacent sides of a parallelogram are represented by the vectors and Find unit vectors parallel to the diagonals of the parallelogram.


Answer:


Side BC parallel to



So resultant vector c = b + a


So, vector




So unit vector along the diagonal of


Parallelogram is






Question 5.

If and find


Answer:

We want to find the magnitude of vector


So,





If a vector is given by then the magnitude of vector is generally denoted by which is equal to






Question 6.

If and the coordinates of P are (1, -1, 2), find the coordinates of Q.


Answer:

Position vector of ‘P’ is


Let the position vector of point ‘Q’ is ‘a.’


So we need to find the value of ‘a.’


Position vector of ‘Q’ - Position vector of ‘P’






So the position vector of ‘Q” is (4, 1, 1)



Question 7.

Prove that the points and are the vertices of a right-angled triangle.


Answer:

In a right angle triangle


Where CA is the hypotenuse


BC is the perpendicular and AB is the base


Vertices of the triangle are given below


A = (1, -1, 0), B = (4, -3, 1), C = (2, -4, 5)


So,







………..(1)


Similarly,






…………(2)






…………..(3)




35 = 14 + 21


35 = 35


LHS = RHS


So, these point form a right angle triangle



Question 8.

If the vertices A, B, C of a triangle ABC are the points with position vectors respectively, what are the vectors determined by its sides? Find the length of these vectors.


Answer:

Let the position vector of the vertex ‘A’ is ,


And similarly B and C


Side AB is



………..(1)


Equation (1) vector representation of the side AB


Magnitude of side AB,



And similarly for side BC and CA



……….(2)




Length of side BC and CA





Question 9.

Find the vector from the origin O to the centroid of the triangle whose vertices are (1, -1, 2), (2, 1, 3) and (-1, 2, -1).


Answer:

Centeroid of the triangle with Vertices (x1, y1, z1), (x2, y2, z2), and (x3, y3, z3) is given by,



In vector algebra, ‘x’ consider as a coefficient of and ‘y’ as a coefficient of and ‘z’ as a coefficient of


So the position vector of the centroid,




So the location of the centroid is


And the vector is,




Question 10.

Find the position vector of a point R which divides the line segment joining points and in the ratio 2 : 1.

(i) Internally

(ii) Externally


Answer:

By using section formula,


(1) Internally


Position vectors of P and Q are given as


and



The position vector of point R which divides the line joining two points P and Q internally in the ratio 2 : 1 is given by,




The position vector of point R which divides the line joining two points P and Q externally in the ratio 2 : 1 is given by,


(2)Externally






Question 11.

Find the position vector of the mid-point of the vector joining the points and


Answer:

If P and Q are two points with position vector and then the position vector of mid point A is given by



Let A is the mid point of PQ.


So, position vector of A







Question 12.

Find the unit vector in the direction of vector where P and Q are the points (1, 2, 3) and (4, 5, 6).


Answer:

First we need to create vector PQ


Position vector of P = OP = (1, 2, 3) and position vector of Q = OQ = (4, 5, 6)





So unit vector in the direction PQ,







Question 13.

Show that the points are the vertices of a right angled triangle.


Answer:

If A, B and C are the vertices of the right angle triangle


So,


In a right angle triangle


Where AB is the hypotenuse


BC is the perpendicular and CA is the base


Vertices of the triangle are given bellow


, and


So,






……(1)


Similarly,






……(2)






……(3)




41 = 35 + 6


41 = 41


LHS = RHS



Question 14.

Find the position vector of the mid-point of the vector joining the points P (2, 3, 4) and Q (4, 1, -2).


Answer:

If P and Q are two points with position vector and then the position vector of mid point A is given by



Let A is the mid point of PQ.


So, position vector of A







Question 15.

Find the value of x for which is a unit vector.


Answer:

We need to find the value of ‘x’ for which is a unit vector


If any vector is a unit vector, then its magnitude should be one.


So, the magnitude of the vector is,






For this value of ‘x’ the above vector is a unit vector



Question 16.

If and find a unit vector parallel to


Answer:

First, we need to create a vector in the direction of


So,




So the unit vector in the direction of is,




This is the unit vector in the direction of



Question 17.

If and find a vector of magnitude 6 units which is parallel to the vector


Answer:

Vector parallel to is,


If a vector parallel to other vector, so we can write a scalar multiple of the other so,


=







So,


=


this is given in the question








So the vector parallel to the is =



Question 18.

Find a vector of magnitude of 5 units parallel to the resultant of the vector and


Answer:



Let resultant vector is ‘R’ so the resultant vector by using the vector triangle law




If a vector parallel to other vector so we can write scalar multiple of the other so,


=


= has a magnitude of 5 unit so







So the vector is =



Question 19.

The two vectors and represent the sides and respectively of triangle ABC. Find the length of the median through A.


Answer:


Let D be the point at which median drawn from A touches side BC.


Let be the position vectors of the vertices A, B and C.


So position vector of D =


So we creating a vector in the direction of AD


Position vector of D – position vector of A







So length of AD







Exercise 23.7
Question 1.

Show that the points A, B, C with position vectors and are collinear.


Answer:

Let us understand that, two more points are said to be collinear if they all lie on a single straight line.

We have been given that,





So, in this case if we prove that and are parallel to each other, then we can easily show that A, B and C are collinear.


Therefore, is given by






And is given by






Let us note the relation between and .


We know,


Or


Or [∵, ]


This relation shows that and are parallel to each other.


But also, is the common vector in and .


and are not parallel but lies on a straight line.


Thus, A, B and C are collinear.



Question 2.

If are non-coplanar vectors, prove that the points having the following position vectors are collinear:



Answer:

Let us understand that, two more points are said to be collinear if they all lie on a single straight line.

Given that, , and are non-coplanar vectors.


And we know that, vectors that do not lie on the same plane or line are called non-coplanar vectors.


To Prove: , and are collinear.


Proof: Let the points be A, B and C.


Then,





So, in this case if we prove that and are parallel to each other, then we can easily show that A, B and C are collinear.


Therefore, is given by





And is given by






Let us note the relation between and .


We know,


Or


Or


Or [∵, ]


This relation shows that and are parallel to each other.


But also, is the common vector in and .


and are not parallel but lies on a straight line.


Thus, A, B and C are collinear.



Question 3.

If are non-coplanar vectors, prove that the points having the following position vectors are collinear:



Answer:

Let us understand that, two more points are said to be collinear if they all lie on a single straight line.

Given that, , and are non-coplanar vectors.


And we know that, vectors that do not lie on the same plane or line are called non-coplanar vectors.


To Prove: , and are collinear.


Proof: Let the points be A, B and C.


Then,





So, in this case if we prove that and are parallel to each other, then we can easily show that A, B and C are collinear.


Therefore, is given by






And is given by






Let us note the relation between and .


We know,


Or


Or [∵, ]


This relation shows that and are parallel to each other.


But also, is the common vector in and .


and are not parallel but lies on a straight line.


Thus, A, B and C are collinear.



Question 4.

Prove that the points having position vectors are collinear.


Answer:

Let us understand that, two more points are said to be collinear if they all lie on a single straight line.


Let the points be A, B and C having position vectors such that,





So, in this case if we prove that and are parallel to each other, then we can easily show that A, B and C are collinear.


Therefore, is given by






And is given by






Let us note the relation between and .


We know,


Or


Or [∵, ]


This relation shows that and are parallel to each other.


But also, is the common vector in and .


and are not parallel but lies on a straight line.


Thus, A, B and C are collinear.



Question 5.

If the points with position vectors and are collinear, find the value of a.


Answer:

Let us understand that, two more points are said to be collinear if they all lie on a single straight line.


Let the points be A, B and C having position vectors such that,





So, let us find and .


Therefore, is given by





…(i)


And is given by





…(ii)


Since, it has been given that points A, B and C are collinear.


So, we can write as



Where λ = a scalar quantity


Put the values of and from (i) and (ii), we get




Comparing the vectors and respectively, we get


a – 12 = 2λ …(iii)


and, 16 = –8λ


From –8λ = 16, we can find the value of λ.


–8λ = 16



⇒ λ = –2


Put λ = –2 in equation (iii), we get


a – 12 = 2λ


⇒ a – 12 = 2(–2)


⇒ a – 12 = –4


⇒ a = –4 + 12


⇒ a = 8


Thus, we have got a = 8.



Question 6.

If are two non-collinear vectors, prove that the points with position vectors and are collinear for all real values of


Answer:

Let us understand that, two more points are said to be collinear if they all lie on a single straight line.


Given that, and are two non-collinear vectors.


Let the points be A, B and C having position vectors such that,





So, in this case if we prove that and are parallel to each other, then we can easily show that A, B and C are collinear.


Therefore, is given by






…(i)


And is given by






Let us note the relation between and .


We know,


Or


Or [∵, from (i)]


Or …(ii)


If λ is any real value, then is also a real value.


Then, for any real value , we can write



From (ii) equation, we can write



This relation shows that and are parallel to each other.


But also, is the common vector in and .


and are not parallel but lies on a straight line.


Thus, A, B and C are collinear.



Question 7.

If prove that A, B, C are collinear points.


Answer:

Let us understand that, two more points are said to be collinear if they all lie on a single straight line.


Given:


To Prove: A, B and C are collinear points.


Proof: We have been given that,



Rearrange it so that we get a relationship between and .




…(i)


Now, we know that



But actually we are doing , such that O is the point of origin so that the difference between the two vectors is a displacement.


So, …(ii)


Similarly,
…(iii)


Substituting equation (ii) & (iii) in equation (i), we get



Thus, this relation shows that and are parallel to each other.


But also, is the common vector in and .


and are not parallel but lies on a straight line.


Hence, A, B and C are collinear.



Question 8.

Show that the vectors and are collinear.


Answer:

Let us understand that, two more points are said to be collinear if they all lie on a single straight line.

We have been given position vectors and .


Let




Also, let O be the initial point having position vector as



Now, let us find and .


is given by





is given by





We have as






[∵, ]


Thus, this relation shows that and are parallel to each other.


But also, is the common vector in and .


and are not parallel but lies on a straight line.


⇒ A and B are collinear.


Hence, and are collinear.



Question 9.

If the points A (m, –1), B (2, 1) and C(4, 5) are collinear, find the value of m.


Answer:

Let us understand that, two more points are said to be collinear if they all lie on a single straight line.


We have been given points:


A (m, –1), B (2, 1) and C (4, 5).


These points are collinear.


Let us define the position vectors as,





Now, we need to find the vectors and .


is given by






And is given by







Since, A, B, C and D are collinear. We can draw a relation between and .



Putting the values of and , we get




Comparing L.H.S and R.H.S, we get


2 – m = 2λ


And 2 = 4λ


We need to find the value of λ in order to find m.


We have


2 = 4λ




Putting the value of λ in equation (2 – m) = 2λ



⇒ 2 – m = 1


⇒ m = 2 – 1


⇒ m = 1


Thus, the value of m = 1.



Question 10.

Show that the points (3, 4), (–5, 16), (5, 1) are collinear.


Answer:

Let us understand that, two more points are said to be collinear if they all lie on a single straight line.


Let the points be A (3, 4), B (–5, 16) and C (5, 1).


Let





So, in this case if we prove that and are parallel to each other, then we can easily show that A, B and C are collinear.


Therefore, is given by







And is given by







Let us note the relation between and .


We know,


Or …(i)


And we know,


Or


Or …(ii)


Substituting the value of in equation (i), we get




This relation shows that and are parallel to each other.


But also, is the common vector in and .


and are not parallel but lies on a straight line.


Thus, A, B and C are collinear.



Question 11.

If the vectors and are collinear, find the value of m.


Answer:

Let us understand that, two more points are said to be collinear if they all lie on a single straight line.


We have the position vectors as,




Since, a and b are collinear. We can draw a relation between and .



Putting the values of and , we get




Comparing L.H.S and R.H.S, we get


2 = –6λ


And –3 = mλ


We need to find the value of λ in order to find m.


We have


2 = –6λ




Putting the value of λ in equation –3 = mλ



⇒ m = 3 × 3


⇒ m = 9


Thus, the value of m = 9.



Question 12.

Show that the points A (1, –2, –8), B (5, 0, –2) and C(11, 3, 7) are collinear, and find the ratio in which B divides AC.


Answer:

We have been given the points A (1, –2, –8), B (5, 0, –2) and C (11, 3, 7).

We need to show that A, B and C are collinear.


Let us define the position vector.





So, in this case if we find a relation between , and , then we can easily show that A, B and C are collinear.


Therefore, is given by







And is given by







And is given by






Let us add and , we get






Thus, clearly A, B and C are collinear.


We need to find the ratio in which B divides AC.


Let the ratio at which B divides AC be λ : 1. Then, position vector of B is:



But the position vector of B is .


So, by comparing the position vectors of B, we can write





Solving these equations separately, we get



⇒ 11λ + 1 = 5(λ + 1)


⇒ 11λ + 1 = 5λ + 5


⇒ 11λ – 5λ = 5 – 1


⇒ 6λ = 4




The ratio at which B divides AC is λ : 1.


Since,


We can say



Solving it further, multiply the ratio by 3.



⇒ λ : 1 = 2 : 3


Thus, the ratio in which B divides AC is 2 : 3.



Question 13.

Using vectors show that the points
A(–2, 3, 5), B(7, 0, 1) C(–3, –2, –5) and D(3, 4, 7) are such that AB and CD intersect at the point P(1, 2, 3).


Answer:

We have been given the points A(–2, 3, 5), B(7, 0, 1), C(–3, –2, –5), D(3, 4, 7) and P(1, 2, 3).

Let us define it position vectors.


So,







Now, we need to show that AB and CD intersect at the point P.


For this, if we prove that A, B and P are collinear & C, D and P are collinear so that P is the common point between them and we can show that AB and CD intersect at P.


Let us find position vector of AP and PB.






And






Now, we can draw out a relation between and .


We know,




This relation clearly shows that and are parallel.


And since, P is the common point between them, we can say that these vectors and are actually not parallel but lie on a straight line.


⇒ Points A, P, B are collinear


[∵, Two more points are said to be collinear if they all lie on a single straight line.]


Now let us find the position vector of CP and PD.






And






Now, we can draw out a relation between and .


We know,




This relation clearly shows that and are parallel.


And since, P is the common point between them, we can say that these vectors and are actually not parallel but lie on a straight line.


⇒ Points C, P and D are collinear.


[∵, Two more points are said to be collinear if they all lie on a single straight line.]


Since, we know that A, P, B and C, P, D are collinear separately.


Note that, P is the common point between the two pairs of collinear points.


Thus, AB and CD intersect each other at a point P.



Question 14.

Using vectors, find the value of such that the points (1, –1, 3) and (3, 5, 3) are collinear.


Answer:

Let the points be A (λ, –10, 3), B (1, –1, 3), C (3, 5, 3).

Let us define the position vectors of A, B and C.





Then,






And






And since, A, B and C are collinear.


Then, it has a relation as such


, where k is scalar quantity.




Comparing the coefficients of and . We get


1 – λ = k (3 – λ)


And 9 = 15k


First, we need to find the value of k.


So take 9 = 15k




Substitute the value of k in (1 – λ) = k (3 – λ)



⇒ 5(1 – λ) = 3(3 – λ)


⇒ 5 – 5λ = 9 – 3λ


⇒ 5λ – 3λ = 5 – 9


⇒ 2λ = –4



⇒ λ = –2


Hence, the value of λ is –2.




Exercise 23.8
Question 1.

Show that the points whose position vectors are as given below are collinear :

and


Answer:

Let us understand that, two more points are said to be collinear if they all lie on a single straight line.

Let us assume points to be A, B and C such that





Then, we need to find and .






And






Now, we need to draw a relation between and .


We know that,



Or


Or


This relation shows that and are parallel to each other.


But since, B is the common point in AB and BC.


⇒ AB and BC actually lies on a straight line.


Thus, A, B and C are collinear.



Question 2.

Show that the points whose position vectors are as given below are collinear :

and


Answer:

Let us assume points to be A, B and C such that





Then, we need to find and .







And






Now, we need to draw a relation between and .


We know that,



Or


Or


This relation shows that and are parallel to each other.


But since, B is the common point in AB and BC.


⇒ AB and BC actually lies on a straight line.


Thus, A, B and C are collinear.



Question 3.

Using vector method, prove that the following points are collinear.

A(6, –7, –1), B(2 –3, 1) and C(4, –5, 0)


Answer:

Let us understand that, two more points are said to be collinear if they all lie on a single straight line.

Given: A (6, –7, –1), B (2, –3, 1) and C (4, –5, 0).


To Prove: A, B and C are collinear.


Proof:


Let us define position vectors. So,





So, in this case if we prove that and are parallel to each other, then we can easily show that A, B and C are collinear.


Therefore, is given by






And is given by






Let us note the relation between and .


We know,


Or


Or [∵, ]


This relation shows that and are parallel to each other.


But also, is the common vector in and .


and are not parallel but lies on a straight line.


Thus, proved that A, B and C are collinear.



Question 4.

Using vector method, prove that the following points are collinear.

A(2, –1, 3), B(4, 3, 1) and C(3, 1, 2)


Answer:

Given: A (2, –1, 3), B (4, 3, 1) and C (3, 1, 2).


To Prove: A, B and C are collinear.


Proof:


Let us define position vectors. So,





So, in this case if we prove that and are parallel to each other, then we can easily show that A, B and C are collinear.


Therefore, is given by






And is given by






Let us note the relation between and .


We know,


Or


Or [∵, ]


This relation shows that and are parallel to each other.


But also, is the common vector in and .


and are not parallel but lies on a straight line.


Thus, proved that A, B and C are collinear.



Question 5.

Using vector method, prove that the following points are collinear.

A(1, 2, 7), B(2, 6, 3) and C(3, 10 –1)


Answer:

Given: A (1, 2, 7), B (2, 6, 3) and C (3, 10, –1).


To Prove: A, B and C are collinear.


Proof:


Let us define position vectors. So,





So, in this case if we prove that and are parallel to each other, then we can easily show that A, B and C are collinear.


Therefore, is given by







And is given by







Let us note the relation between and .


We know,


Or


This relation shows that and are parallel to each other.


But also, is the common vector in and .


and are not parallel but lies on a straight line.


Thus, proved that A, B and C are collinear.



Question 6.

Using vector method, prove that the following points are collinear.

A(–3, –2, –5), B(1, 2, 3) and C(3, 4, 7)


Answer:

Given: A (–3, –2, –5), B (1, 2, 3) and C (3, 4, 7).


To Prove: A, B and C are collinear.


Proof:


Let us define position vectors. So,





So, in this case if we prove that and are parallel to each other, then we can easily show that A, B and C are collinear.


Therefore, is given by







And is given by







Let us note the relation between and .


We know,


Or


Or [∵, ]


This relation shows that and are parallel to each other.


But also, is the common vector in and .


and are not parallel but lies on a straight line.


Thus, proved that A, B and C are collinear.



Question 7.

Using vector method, prove that the following points are collinear.

A (2, –1, 3), B (3, –5, 1) and C (–1, 11, 9).


Answer:

A (2, –1, 3), B (3, –5, 1) and C (–1, 11, 9).


To Prove: A, B and C are collinear.


Proof:


Let us define position vectors. So,





So, in this case if we prove that and are parallel to each other, then we can easily show that A, B and C are collinear.


Therefore, is given by







And is given by







Let us note the relation between and .


We know,


Or


Or [∵,


This relation shows that and are parallel to each other.


But also, is the common vector in and .


and are not parallel but lies on a straight line.


Thus, proved that A, B and C are collinear.



Question 8.

If are non–zero, non-coplanar vectors, prove that the following vectors are coplanar :

and


Answer:

Vectors parallel to the same plane, or lie on the same plane are called coplanar vectors

The three vectors are coplanar if one of them is expressible as a linear combination of the other two.


We have been given that, , and .


We can form a relation using these three vectors. Say,





Compare the vectors , and . We get


5 = 7x + 3y …(1)


6 = –8x + 20y …(2)


7 = 9x + 5y …(3)


Solving equations (1) and (2) for x and y.


Equation (1), 7x + 3y = 5


Equation (2), –8x + 20y = 6


Multiply equation (1) by 8 and equation (2) by 7, we get


7x + 3y = 5 [× 8


–8x + 20y = 6 [× 7


We get



⇒ 164y = 82





Put in equation (2), we get



⇒ –8x + 10 = 6


⇒ –8x = 6 – 10


⇒ –8x = –4


⇒ 8x = 4




Substituting and in equation (3), we get


7 = 9x + 5y


Or 9x + 5y = 7





⇒ 14 = 7 × 2


⇒ 14 = 14


∵, L.H.S = R.H.S


⇒ The value of x and y satisfy equation (3).


Thus, , and are coplanar.



Question 9.

If are non–zero, non-coplanar vectors, prove that the following vectors are coplanar :

and


Answer:

We have been given that, , and .

We can form a relation using these three vectors. Say,





Compare the vectors , and . We get


1 = – 2y …(1)


–2 = –3x + 3y …(2)


3 = 5x – 4y …(3)


Solving equation (1) for y,


Equation (1), –2y = 1



Put in equation (2), we get





⇒ –6x – 3 = –2 × 2


⇒ –6x – 3 = –4


⇒ –6x = –4 + 3


⇒ –6x = –1



Substituting and in equation (3), we get


3 = 5x – 4y


Or 5x – 4y = 3







But


∵, L.H.S ≠ R.H.S


⇒ The value of x and y doesn’t satisfy equation (3).


Thus, , and are not coplanar.



Question 10.

Show that the four points having position vectors are coplanar.


Answer:

Vectors parallel to the same plane, or lie on the same plane are called coplanar vectors

The three vectors are coplanar if one of them is expressible as a linear combination of the other two.


Let the four points be denoted be P, Q, R and S for , , and respectively such that we can say,






Let us find , and .


So,







Also,







And,







Now, we need to show a relation between , and .


So,





Comparing coefficients of , and , we get


–6x – 4y = 10 …(i)


10x + 2y = –12 …(ii)


–6x + 10y = –4 …(iii)


For solving equation (i) and (ii) for x and y, multiply equation (ii) by 2.


10x + 2y = –12 [× 2


⇒ 20x + 4y = –24 …(iv)


Solving equations (iv) and (i), we get



⇒ 14x = –14



⇒ x = –1


Put x = –1 in equation (i), we get


–6(–1) – 4y = 10


⇒ 6 – 4y = 10


⇒ –4y = 10 – 6


⇒ –4y = 4



⇒ y = –1


Substitute x = –1 and y = –1 in equation (iii), we get


–6x + 10y = –4


⇒ –6(–1) + 10(–1) = –4


⇒ 6 – 10 = –4


⇒ –4 = –4


∵, L.H.S = R.H.S


⇒ The value of x and y satisfy equation (iii).


Thus, , , and are coplanar.



Question 11.

Prove that the following vectors are coplanar :

and


Answer:

Vectors parallel to the same plane, or lie on the same plane are called coplanar vectors

The three vectors are coplanar if one of them is expressible as a linear combination of the other two.


We have been given that, , and .


We can form a relation using these three vectors. Say,





Comparing coefficients of , and , we get


2 = x + 3y …(1)


–1 = –3x – 4y …(2)


1 = –5x – 4y …(3)


Solving equations (1) and (2) for x and y.


Equation (1), x + 3y = 2


Equation (2), –3x – 4y = –1


Multiply equation (1) by 3.


x + 3y = 2 [× 3


⇒ 3x + 9y = 6 …(4)


Solving equations (4) and (2), we get



⇒ 5y = 5



⇒ y = 1


Put in equation (1), we get


2 = x + 3y


⇒ x + 3(1) = 2


⇒ x = 2 – 3


⇒ x = –1


Substituting x = –1 and y = 1 in equation (3), we get


–5x – 4y = 1


⇒ –5(–1) – 4(1) = 1


⇒ 5 – 4 = 1


⇒ 1 = 1


∵, L.H.S = R.H.S


⇒ The value of x and y satisfy equation (3).


Thus, , and are coplanar.



Question 12.

Prove that the following vectors are coplanar :

and


Answer:

We have been given that, , and .

We can form a relation using these three vectors. Say,





Comparing coefficients of , and , we get


1 = 2x – y …(1)


1 = 3x – 2y …(2)


1 = –x + 2y …(3)


Solving equations (1) and (2) for x and y.


Equation (1), 2x – y = 1


Equation (2), 3x – 2y = 1


Multiply equation (1) by 2.


2x – y = 1 [× 2


⇒ 4x – 2y = 2 …(4)


Solving equations (4) and (2), we get



⇒ x = 1


Put x = 1 in equation (1), we get


1 = 2x – y


⇒ 1 = 2(1) – y


⇒ 1 = 2 – y


⇒ y = 2 – 1


⇒ y = 1


Substituting x = 1 and y = 1 in equation (3), we get


1 = –x + 2y


Or –x + 2y = 1


⇒ –(1) + 2(1) = 1


⇒ –1 + 2 = 1


⇒ 1 = 1


∵, L.H.S = R.H.S


⇒ The value of x and y satisfy equation (3).


Thus, , and are coplanar.



Question 13.

Prove that the following vectors are non-coplanar :

and


Answer:

Vectors parallel to the same plane, or lie on the same plane are called coplanar vectors

The three vectors are coplanar if one of them is expressible as a linear combination of the other two.


We have been given that, , and .


We can form a relation using these three vectors. Say,





Comparing coefficients of , and , we get


3 = 2x + 7y …(1)


1 = –x – y …(2)


–1 = 7x + 23y …(3)


Solving equations (1) and (2) for x and y.


Equation (1), 2x + 7y = 3


Equation (2), –x – y = 1


Multiply equation (2) by 2.


–x – y = 1 [× 2


⇒ –2x – 2y = 2 …(4)


Solving equations (4) and (1), we get



⇒ 5y = 5



⇒ y = 1


Put y = 1 in equation (2), we get


1 = –x – y


⇒ 1 = –x – (1)


⇒ 1 = –x – 1


⇒ x = –1 – 1


⇒ x = –2


Substituting x = –2 and y = 1 in equation (3), we get


–1 = 7x + 23y


Or 7x + 23y = –1


⇒ 7(–2) + 23(1) = –1


⇒ –14 + 23 = –1


⇒ 9 ≠ –1


∵, L.H.S ≠ R.H.S


⇒ The value of x and y doesn’t satisfy equation (3).


Thus, , and are not coplanar.



Question 14.

Prove that the following vectors are non–coplanar :

and


Answer:

We have been given that, , and .

We can form a relation using these three vectors. Say,





Comparing coefficients of , and , we get


1 = 2x + y …(1)


2 = x + y …(2)


3 = 3x + y …(3)


Solving equations (1) and (2) for x and y.


Equation (1), 2x + y = 1


Equation (2), x + y = 2



⇒ x = –1


Put x = –1 in equation (2), we get


2 = x + y


⇒ 2 = (–1) + y


⇒ y = 2 + 1


⇒ y = 3


Substituting x = –1 and y = 3 in equation (3), we get


3 = 3x + y


Or 3x + y = 3


⇒ 3(–1) + (3) = 3


⇒ –3 + 3 = 3


⇒ 0 ≠ 3


∵, L.H.S ≠ R.H.S


⇒ The value of x and y doesn’t satisfy equation (3).


Thus, , and are not coplanar.



Question 15.

If are non–coplanar vectors, prove that the following vectors are non–coplanar :

and


Answer:

Vectors parallel to the same plane, or lie on the same plane are called coplanar vectors

The three vectors are coplanar if one of them is expressible as a linear combination of the other two.


We have been given that, , and .


We can form a relation using these three vectors. Say,





Compare the vectors , and . We get


2 = x + y …(1)


–1 = x + y …(2)


3 = –2x – 3y …(3)


Solving equations (1) and (2) for x and y.


Equation (1), x + y = 2


Equation (2), x + y = –1


We get



The value of x and y cannot be found so it won’t satisfy equation (3).


Thus, , and are not coplanar.



Question 16.

If are non–coplanar vectors, prove that the following vectors are non-coplanar :

and


Answer:

We have been given that, , and .

We can form a relation using these three vectors. Say,





Compare the vectors , and . We get


1 = 2x + y …(1)


2 = x + y …(2)


3 = 3x + y …(3)


Solving equation (1) and (2) for x and y,



⇒ x = –1


Put x = –1 in equation (2), we get


⇒ 2 = x + y


⇒ 2 = –1 + y


⇒ y = 2 + 1


⇒ y = 3


Substituting x = –1 and y = 3 in equation (3), we get


3 = 3x + y


Or 3x + y = 3


⇒ 3(–1) + 3 = 3


⇒ –3 + 3 = 3


⇒ 0 ≠ 3


∵, L.H.S ≠ R.H.S


⇒ The value of x and y doesn’t satisfy equation (3).


Thus, , and are not coplanar.



Question 17.

Show that the vectors given
by and are non-coplanar. Express vector as a linear combination of the vectors and


Answer:

Vectors parallel to the same plane, or lie on the same plane are called coplanar vectors

The three vectors are coplanar if one of them is expressible as a linear combination of the other two.


Given that





Let






Comparing the coefficients of , and , we get


1 = 2x + y …(1)


2 = x + y …(2)


3 = 3x + y …(3)


Solving equation (1) and (2), we get



⇒ x = –1


Substitute x = –1 in equation (2), we get


2 = x + y


⇒ 2 = –1 + y


⇒ y = 2 + 1


⇒ y = 3


Put x = –1 and y = 3 in equation (3), we get


3 = 3x + y


⇒ 3 = 3(–1) + 3


⇒ 3 = –3 + 3


⇒ 3 ≠ 0


∴ L.H.S ≠ R.H.S


⇒ The value of x and y doesn’t satisfy equation (3).


Thus, , and are not coplanar.


Let be depicted as,


…(*)


Substitute the value of , , and .





Comparing the coefficients in , and , we get


2 = x + 2y + z …(1)


–1 = 2x + y + z …(2)


–3 = 3x + 3y + z …(3)


From equation (1),


2 = x + 2y + z


⇒ z = 2 – x – 2y …(4)


Putting the value of z from equation (4) in equations (2) & (3), we get


From equation (2),


–1 = 2x + y + z


⇒ –1 = 2x + y + (2 – x – 2y)


⇒ –1 = 2x + y + 2 – x – 2y


⇒ 2x – x + y – 2y = –1 – 2


⇒ x – y = –3 …(5)


From equation (3),


–3 = 3x + 3y + z


⇒ –3 = 3x + 3y + (2 – x – 2y)


⇒ –3 = 3x + 3y + 2 – x – 2y


⇒ 3x – x + 3y – 2y = –3 – 2


⇒ 2x + y = –5 …(6)


Solving equation (5) and (6), we have



⇒ 3x = –8



Substituting in equation (5), we get


x – y = –3




⇒ –8 – 3y = –3 × 3


⇒ –8 – 3y = –9


⇒ 3y = 9 – 8


⇒ 3y = 1



Now, substitute and in z = 2 – x – 2y, we get








⇒ z = 4


We have got , and z = 4.


Put these values in equation (*), we get



Thus, we have found the relation.



Question 18.

Prove that a necessary and sufficient condition for three vectors and to be coplanar is that there exist scalars not all zero simultaneously such that


Answer:

Given: The vectors , and .


To Prove: (a). Necessary condition: The vectors , and will be coplanar if there exist scalar l, m, n not all zero simultaneously such that .


(b). Sufficient condition: For vectors , and , there exist scalar l, m, n not all zero simultaneously such that


Proof:


(a). Necessary condition: Let , and are three coplanar vectors.


Then, one of them can be expressed as a linear combination of the other two.


Then, let


Rearranging them we get,



Here, let


x = l


y = m


–1 = n


We have,



Thus, if , and are coplanars, there exists scalar l, m and n (not all zero simultaneously zero) such that .


∴ necessary condition is proved.


(b). Sufficient condition: Let , and be three vectors such that there exists scalars l, m and n not all simultaneously zero such that .




Now, divide by n on both sides, we get




Here, we can see that


is the linear combination of and .


⇒ Clearly, , and are coplanar.


∴ sufficient condition is also proved.


Hence, proved.



Question 19.

Show that the four points A, B, C and D with position vectors and respectively are coplanar if and only if


Answer:

Given that,






Let A, B, C and D be coplanar.


As we know that, the vectors , , and will be coplanar if there exist scalar x, y, z, u not all zero simultaneously such that .


Then, we can write



Where, (x + y + z + u) = 0


Provided x, y, z, u are scalars not all simultaneously zero.


Let x = 3, y = –2, z = 1 and u = –2


So, we get



Thus, A, B, C and D are coplanar if .



If is true.


Rearranging it, we get



Dividing this from the sum of its coefficient (that is, 4) on both sides,



Or


⇒ There is a point say P, which divides the line AC in ratio 1:3 and BD in ratio 2:2 internally.


Thus, P is the point of interaction of AC and BD.


As, vectors parallel to the same plane, or lie on the same plane are called coplanar vectors.


Hence, A, B, C and D are coplanar.




Exercise 23.9
Question 1.

Can a vector have direction angles 45o, 60o, 120o.


Answer:

We know that, If l, m, n are the direction cosine of a vector and α, β, γ are the direction angle, then –

l = cosα, m = cosβ, n = cosγ


And, l2 + m2 + n2 = 1 …… (i)


∴ l = cos45°, m = cos60°, n = cos120°



Now, substituting l, m, n in equation (i), we get -






⇒ 1 = 1


⇒ L.H.S = R.H.S


∴ A vector can have direction angles 45o, 60o, 120o.



Question 2.

Prove that 1, 1, and 1 cannot be direction cosines of a straight line.


Answer:

Here, l = 1, m = 1, n = 1

And, we know that –


l2 + m2 + n2 = 1


Taking LHS,


l2 + m2 + n2 = (1)2 + (1)2 + (1)2


= 3


≠1


⇒ LHS≠RHS


∴ 1, 1, and 1 cannot be direction cosines of a straight line.



Question 3.

A vector makes an angle of π/4 with each of x - axis and y - axis. Find the angle made by it with the z - axis.


Answer:

Given,

l = cosα




m = cosα




And, n = cosγ


Also,


l2 + m2 + n2 = 1




⇒ 1 + cos2γ = 1


⇒ cos2γ = 0


⇒ cosγ = 0


⇒ γ = cos-10



The angle made by the vector with the z - axis =



Question 4.

A vector is inclined at equal acute angles to x – axis, y – axis, and z - axis. If units, find


Answer:

Here, α = β = γ

⇒ cosα = cosβ = cosγ


⇒ l = m = n = p(say)


Now, we know that -


l2 + m2 + n2 = 1


⇒ p2 + p2 + p2 = 1


⇒ 3p2 = 1



∴ the direction cosines of r⃗ are -



⇒ r⃗ = |r⃗|( l î + m ĵ + n k̂)




Now, multiplying and dividing it by √3



⇒ r⃗ = ±2√ 3 ( î + ĵ + k̂)



Question 5.

A vector is inclined to the x - axis at 45o and y - axis at 60o. If units, find


Answer:

Here, α = 45°,β = 60°,γ = θ (say)

l = cosα


= cos45°



m = cosα


= cos45°



n = cosθ


Now, substituting l, m,n in


l2 + m2 + n2 = 1,








∴ r⃗ = |r⃗ |( l î + m ĵ + n k̂)




⇒ r⃗ = 4(√2 î + ĵ± k̂)



Question 6.

Find the direction cosines of the following vectors :

i.

ii.

iii.


Answer:

(i)

Here,


The direction ratios of the vector 2 î + 2 j - k̂ are 2, 2, - 1


The direction cosines of the vector





∴ The direction cosines of are given by


(ii)


Here,


The direction ratios of the vector 6 î - 2 j - 3 k̂ are 6, - 2, - 3


The direction cosines of the vector





∴ The direction cosines of are given by


(iii)


Here,


The direction ratios of the vector 3 î - 4k̂ are 3, 0, - 4


The direction cosines of the vector





∴ The direction cosines of are given by



Question 7.

Find the angles at which the following vectors are inclined to each of the coordinate axes :

i.

ii.

iii.


Answer:

(i)

Let, r⃗ = î - j + k̂


The direction ratios of the vector r⃗ = 1, - 1, 1


And, |r⃗ | = √ ((1)2 + ( - 1)2 + (1)2)


= √3


The direction cosines of the vector r⃗



So,








Thus, angles made by with the coordinate axes are given by


(ii)


Let, r⃗ = 0 î + j - k̂


The direction ratios of the vector r⃗ = 0, 1, - 1


And, |r⃗ | = √ ((0)2 + (1)2 + ( - 1)2)


= √2


The direction cosines of the vector r⃗



So,












Thus, angles made by with the coordinate axes are given by


(iii)


Let, r⃗ = 4î + 8 j + k̂


The direction ratios of the vector r⃗ = 4, 8, 1


And, |r⃗ | = √ ((4)2 + (8)2 + (1)2)


= √81


= 9


The direction cosines of the vector r⃗



So,








Thus, angles made by with the coordinate axes are given by



Question 8.

Show that the vector is equally inclined with the axes OX, OY and OZ.


Answer:

Let r⃗ = î + j + k̂

And, |r⃗ | = √ ((1)2 + (1)2 + (1)2)


= √ 3


Therefore, The direction cosines of the vector r⃗



Now, let α, β and γ be the angles formed by r⃗ with the positive directions of x, y and z axes.


Then,


We have,



Hence, the given vector is equally inclined to axes OX, OY and OZ.



Question 9.

Show that the direction cosines of a vector equally inclined to the axes OX, OY and OZ are


Answer:

Let a vector be equally inclined to axes OX, OY, and OZ at angle

Then, the direction cosines of the vector are l = cosα, m = cosα and n = cos α


And, we know that –


l2 + m2 + n2 = 1


cos2α + cos2α + cos2α = 1


3 cos2α = 1



Hence, the direction cosines of the vector which are equally inclined to the axes are



Question 10.

If a unit vector makes an angle with with and an acute angle θ with then find θ and hence, the components of


Answer:

Let unit vector r⃗ have (r1, r2, r3) components.

⇒ r⃗ = r1î + r2 j + r3


Since, r⃗ is a unit vector.


⇒ |r⃗ | = 1


Also, given that r⃗ makes angles with î,with ĵ and an acute angle θ with k̂


Then, we have:



(∵ |r⃗ | = 1)



(∵ |r⃗ | = 1)


Also,


⇒ cosθ = r3 (∵ |r⃗ | = 1)


Now, |r⃗ | = 1


⇒ √ (r12 + r22 + r32) = 1










and the components of r⃗ are -



Question 11.

Find a vector of magnitude units which makes an angle of and with y and z - axes respectively.


Answer:

Let l, m, n be the direction cosines of the vector r⃗

l = cosα,


m = cosβ



,


And, n = cosγ



= 0


Also, l2 + m2 + n2 = 1




∴ r⃗ = |r⃗ |( l î + m ĵ + n k̂)



(Given, |r⃗ | = 3√ 2 )


⇒ r⃗ = ± 3 î + 3 ĵ



Question 12.

A vector is inclined at equal angles to the three axes. If the magnitude of is find


Answer:

Let l, m, n be the direction cosines of the vector r⃗

Vector r⃗ is inclined at equal angles to the three axes.


l = cosα, m = cos α, n = cosα


⇒ l = m = n.


Also, we know that -


l2 + m2 + n2 = 1


cos2α + cos2α + cos2α = 1


3 cos2α = 1



Hence, the direction cosines of the vector which are equally inclined to the axes are


∴ r⃗ = |r⃗ |( l î + m ĵ + n k̂)



(Given, |r⃗ | = 2√ 3 )


⇒ r⃗ = ±(2 î + 2ĵ + 2 k̂)